You are on page 1of 148

C URRENT E LECTRICITY

TYPES OF MATERIALS
Natural and artificially made materials show a wide range of

electrical properties. The properties are determined partly by the behavior of

individual atoms or molecules and partly by the interactions of atoms or

molecules in the bulk material. The ability of a material to conduct

electricity may also depend on the conditions of the material, such as its

temperature and pressure.

1) Conductors:
Conductors (for example, most metals) are materials in which electric charge readily
flow. In many metals, each atom gives up one or more of its outer or valence electrons to the
entire material, and we often regard the electrons as forming a “gas” within the material
rather than belonging to any particular atom. These electrons are free to move when an
electric field is applied to the material. Under static conditions the electric field in the interior
of a conductor is zero, even if the conductor carries a net charge. (If this were not so, the free
electrons would be accelerated, which would violate the assumption of a static charge
distribution).

2) Insulators:
In an insulator, on the other hand, the electrons are bound rather tightly to the atoms
and are not free to move under the electric fields that might be applied under ordinary
circumstances. An insulator can carry any distribution of electric charges on its surface or in
its interior, and (in contrast to a conductor) the electric field in the interior of an insulator
can have nonzero values.

Ordinary matter is usually electrically neutral. In the absence of an external electric


field, this neutrality applies to individual atoms as well as to the entire material. The
application of an electric field can remove one or more electrons from atoms of the material.
This process is called ionization, and the resulting positively charged atoms with a deficiency
of electrons are called ions. In an insulator, a sufficiently large electric field can ionize the
atoms, and as a result there are electrons available to move through the material. Under
these circumstances an insulator can behave more like a conductor. This situation is called
6 7
breakdown and requires field typically in the range of 10 V /m in air to 10 V /m in plastics
and ceramics.
CURRENT ELECTRICITY 2

3) Semi-conductors:
Intermediates between insulators and conductors are semiconductors. In a
10 12
Semi-conductor, perhaps one atom in 10 to 10 might contribute an electron to the flow of
electricity in the material (in contrast to a conductor, in which every atom typically
contributes an electron to the flow of electricity). Commonly used semiconductors include
silicon and germanium, as well as many compounds.

4) Super conductors:
Even the best conductors (copper, silver, and gold) show a small but definitely nonzero
resistance to the flow of electricity. Under certain conditions, often involving cooling to very
low temperatures, electric charge can flow through some materials with no resistance at all.
This property of materials is called superconductivity, and the materials under these
conditions are called superconductors. Some materials can be relatively poor conductors at
room temperature but can be superconductors at low temperature.

In this chapter we study ways in which conductors and insulators respond to applied
electric fields.

A Conductor In An Electric Field: Static Conditions


Suppose we place a large rectangular slab of a conductor such as copper in a uniform
electric field, as shown in figure 1(a). We can regard the copper as a “gas” of electrons that

are free to move in a lattice of copper ions in fixed locations. The electric field E0 exerts a
 
force F = −eE0 on the electrons, which causes the electrons to move in a direction opposite to
the field. The electrons quickly move to the top surface of the copper, leaving a deficiency of
electrons (a positive charge) on the bottom surface. When we place a conductor in an external
field, the charges redistribute themselves almost instantaneously, after which electrostatic
conditions apply.

The two surfaces of the conductor can be considered as sheets of charge, which set up
 
an electric field E ' as shown in figure 1(b). Inside the coper, the net electric field E is the
  
vector sum of the two fields: = '
E E0 + E . In terms of magnitudes, the sum becomes a
3 CURRENT ELECTRICITY

difference, because the two fields are in opposite directions: = '


E E0 − E . In the interior of the
copper under static conditions, the net electric field E must be zero. (The electric field inside
the conductor must be zero, because otherwise the free electrons in the conductor would be
accelerated, thus violating our assumption of a static situation.) The applied electric field E0
must move just enough electrons to the surface to set up an electric field E ' that has the
same magnitude as E0 , giving a net field of zero inside the copper. Outside the slab, the
sheets of charge on the two surfaces give electric fields that cancel, leaving the net field
unchanged in those regions.
A Conductor In An Electric Field: Dynamic Conditions
In figure 1(a), electrons move from the bottom of the slab of copper to the top under
the action of the applied electric field, until the concentration of electrons at the top of the
slab (and of positive ions at the bottoms) creates a field that cancels the applied field in the
interior of the copper and prevents the flow of additional electrons. Suppose there were a
mechanism to remove electrons from the top of the slab, carry them around an external path,
and re-inject them at the bottom of the slab (shown in figure (2)). In this case, there would be
no build-up of charge on the top and bottom of the slab, and the electrostatic conditions of
the previous section cannot be applied to the copper. In particular, the conclusion drawn in
the last section is no longer valid –– the electric field inside the copper will in general be
nonzero when charges are flowing.

The continuous loop of flowing electrons is a simple representation of an electric circuit, and
the flow of electrons (or other charged particles) is called an electric current.

Electric Current
When there is a transfer of charge from one side of an area to the other, we say that
there is an electric current through the area. If the moving charges are positive, the current
is in the direction of motion. If they are negative, the current is opposite to the direction of
motion. If a charge ∆Q crosses an area in time ∆t , we define the average electric current
through the area during this time as
∆Q
i =
∆t
∆Q dQ
The current at time t is
= =
i lim ………………(i)
∆t →0 ∆t dt
Thus, electric current through an area is the rate of transfer of charge from one side of the
area to the other. The SI unit of current is ampere. If one coulomb of charge crosses an area
in one second, the current is one ampere. It is one of the seven base units accepted in SI.
CURRENT ELECTRICITY 4

Note:
1) Current is a scalar quantity. We often represent a current with an arrow to indicate
that charge is moving. Such arrows are not vectors, however, and they do not require
vector addition. Figure shows a conductor with current i0 splitting at a junction into
two branches. Because charge is conserved, the magnitudes of the currents in the
branches must add to yields the magnitude of the current in the original conductor, so
that
i0= i1 + i2
Current arrows show only a direction (or sense) of flow along a conductor, not a
direction in space.

2) The particles flowing through a surface can be charged positively or negatively, or we


can have two or more types of particle moving, with charges of both the signs in the
flow. Conventionally, we define the direction of the current as direction of flow of positive
charges. In a common conductor such as copper, the current is in physical state due
to the motion of the negatively charged electrons. Therefore, when we speak of current
is such a conductor, the direction of the current is opposite to the direction of flow of
electrons. On the other hand, if one consider a beam of positively charged proton in a
particle accelerator, then the current is in the direction of the motion of the protons. In
some cases, gases and electrolytes, for example, the current is the result of the flow of
both positive and negative charges. It is common to refer to a moving charged particle
(whether it is positive or negative) as a mobile charge carrier. For example, the charge
carriers in a metal are electrons.

3) Current in different situations is defined as follows:


i) If n particles, each having a charge q, pass through a given area in time t, the
current is given by
∆Q nq
=I =
∆t t
ii) If n particles (each having a charge q) pass per second per unit area, the current
associated with cross-sectional area A is
∆Q
=I = nqA
∆t
iii) If there are n particle per unit volume in the conductor (each having a charge q)
moving with velocity v, then current through cross-sectional area A is
∆Q ∆x
=I = nqA = nqvA
∆t ∆t
5 CURRENT ELECTRICITY
iv) If a point charge q is moving in a circle of radius r with speed v, then its time period
 2πr 
T=  . So through a given hypothetical cross-section
 v 
(perpendicular to motion), the current is
q q qv qω
=
I = = =
t T 2πr 2π

Where ω is the angular velocity of the charge.


v) It is possible that a charged body is transported from one place to another. The
convectional current is the current which is developed due to the transportation of
charge or the mechanical transfer of a charge.

4) If current is passing through a wire of non-uniform cross section, then current


through all the cross sections of the wire is same in study state.

= bb
In study state, current through aa '
= cc
= i. ' '
5) Direct current (DC): In this direction of current is constant with time.
Alternating current (AC): In this both magnitude and direction of current changes with
time.
Pulsating (DC) Constant (DC) Alternating current (AC)
i i i
+

t – t
t

Current Density
Sometimes we are interested in the current i in a particular conductor. At other times
we take a localized view and study the flow of charge through a cross section of the

conductor at a particular point. To describe this flow, we can use the current density j ,
which has the same direction as the velocity of the moving charges if they are positive and
the opposite direction if they are negative.

To define the current density at a point P, we draw a small area ∆S through P perpendicular
to the flow of charges (figure 32.1a). If ∆i be the current through the area ∆S , the average
current density is
∆i
j =
∆S
CURRENT ELECTRICITY 6

The current density at the point P is


∆i di
= =
j lim
∆S→0 ∆S dS
The direction of the current density is the same as the direction of the current. If a current i
is uniformly distributed over an area S and is perpendicular to it,
i
j=
S

Now let us consider an area ∆S which is not necessarily perpendicular to the current figure.
If the normal to the area makes an angle θ with the direction of the current, the current
density is,
∆i
j= or, ∆i = j ∆Scos θ
∆Scos θ

Where ∆i is the current through ∆S . If ∆S be the area vector corresponding to the area ∆S ,
we have
 
∆i = j. ∆S
 
For a finite area, i = ∫ j. dS
Note carefully that an electric current has direction as well as magnitude but it is not a
vector quantity. It does not add like vectors. The current density is a vector quantity.

Current Density And Drift Speed



Electrons inside a metal are accelerated by electric field which exerts a force eE on the
electrons. The electrons collide with the ions of the lattice and transfer energy to them. The
motion of individual electrons is therefore very irregular, consisting of a short interval of
acceleration in a direction opposite to the electric field, followed by a collision with an ion
that might send the electron into motion in any direction, followed by another acceleration,
and so on. The net effect is a drift of electrons in a direction opposite to the field. There is no
net acceleration of electrons, because they continually lose energy in collisions with the
lattice of copper ions. In effect, energy is transferred from the applied field to the lattice (in
the form of internal energy of the conductor, often observed as a temperature increase). On

the average, electrons can be described as moving with a constant drift velocity v d in a
direction opposite to the field, as indicated in figure.
7 CURRENT ELECTRICITY

Consider the motion of electrons in a portion of the conductor of length L. The


electrons are moving with drift speed v d , so they travel the length L in a time t = L / v d . The
conductor has a cross-sectional area A, so in the time t all of the electrons in the volume AL
will travel through a surface at the right end of the conductor. If the density of electrons
(number per unit volume) is n, then the magnitude of the net charge passing through the
q enAL
surface is q = enAL , and the current density is =j = = env d
At AL / v d
In vector notation, this is
 
j = −e nv d
The negative sign again remind us that the direction of the current density is opposite to the
motion of the electrons.

The drift speeds of electrons in typical materials are very small compared with the
6
speed of the random thermal motions of electrons (typically 10 m /s ).

LEARN THE SKILLS


1. In a hydrogen discharge tube, the number of protons drifting across a cross section per
second is 1.0 × 1018 , while the number of electrons drifting in the opposite direction across
the same cross section is 2.7 × 1018 per second. Find the current flowing in the tube.
Sol. As electrons and protons are moving in the opposite directions, they will effectively produce
current in the same direction and the total current in the tube is
=I ( np + ne ) e / t = (1.0 ×1018 + 2.7 ×1018 ) ×1.6 ×10−19 /1 = 3.7 × 1.6 × 10−1 A = 0.592 A

2. An electron beam has an aperture 1.0 mm2 . A total of 6.0 × 1016 electrons go through any
perpendicular cross-section per second. Find (a) the current and (b) the current density in
the beam.
Sol. a) The total charge crossing a perpendicular cross-section in one second is
q = ne = 6.0 × 10
16
× 1.6 × 10−19 =
C 9.6 × 10−3 C
q 9.6 × 10−3C
The current is i = = = 9.6 × 10−3 A
t 1s
As the charge is negative, the current is opposite to the direction of motion of the beam.
i 9.6 × 10−3 A
b) The current density is =
j = = 9.6 × 103 A / m2
S 1.0 mm2

3. The current in a wire varies with time according to the relation I= a + bt2 , where current I is
in ampere and time t is in second and a=4 A, b=2 As −2 .
CURRENT ELECTRICITY 8

a) How many coulombs pass a cross-section of the wire in the time interval between t=5 s
and t=10 s?
b) What constant current could transport the same charge in same time interval?
10 10 10
2 3 2
Sol. a) ∆=
q = ∫ ( 4 + 2t
∫ I dt
2
) dt ⇒ ∆q= 4t + t = 4 (10 − 5 ) + (1000 − 25 ) = 603.33 C
3 5 3
5 5
∆q 603.33
b) =
Ie = = 120.67 A
∆t 10 − 5
4. In the Bohr model of hydrogen atom, the electron is pictured to rotate in a circular orbit of
radius 5 × 10−11 m , at a speed of 2.2 × 106 ms −1 . What is the current associated with electron
motion?
2πr
Sol. The time taken to complete one rotation is T =
v
q e ev 1.6 × 10−19 × 2.2 × 106
Therefore, the current is =
I = = = = 1.12 mA
t T 2πr 2 × 3.14 × 5 × 10−11

5. A long cylinder with uniformly charged surface and cross-sectional radius a=1 cm moves
with a constant velocity v = 10 ms −1 along its axis. An electric field strength at the surface of
the cylinder is equal to E. Find the resulting conventional current, that is, the current caused
by mechanical transfer of a charge.
dq
Sol. The convection current is I = …………(i)
dt
where, dq = λdx, λ is the linear charge density
λ
But, from the Gauss’s law, electric field at the surface of the cylinder. E =
2πε0a
Substituting the value of λ and subsequently of dq in equation (i), we get
dq λdx 2Eπε adx  dx 
= I = ⇒ I= 0 ⇒ I= 2πε0 Eav  = v
dt dt dt  dt 
=
Substituting the given values, we get I 0.5 µA

6. One end of an aluminum wire whose diameter is 2.5 mm is welded to one end of a copper
wire whose diameter is 1.8 mm. The composite wire carries a steady current i of 1.3 A. What
is the current density in each wire?
Sol. We may take the current density as (a different) constant within each wire except for points
near the junction. The current density is given by j = i / A . The cross-sectional area A of the
aluminum wire is
1
( )
2
A A1 = πd2 =π ( / 4 ) 2.5 × 10−3 m =4.91 × 10−6 m2 , so that
4
1.3 A
jA1 = −6 2 = 2.6 × 105 A / m2 =
26 A /cm2 .
4.91 × 10 m
As you can verify, the cross-sectional area of the copper wire is 2.54 × 10−6 m2 , so that
1.3A
jCu = 5.1 × 105 A / m2 =
= 51A / cm2
2.54 × j10−6 m
The fact that the wires are of different materials does not enter here.
9 CURRENT ELECTRICITY

7. What is the drift speed of the conduction electrons in the copper wire of the above problem?
Sol. In copper, there is very nearly one conduction electron per atom on the average. The number
n of electrons per unit volume is therefore the same as the number of atoms per unit volume
and is determined from
n ρm atoms / m3 mass / m3
= = or
NA M atoms / mol mass / mol
Here ρm is the (mass) density of copper, N A is the Avogadro constant, and M is the molar
mass of copper. Thus

=
N A ρm
n =
( 6.02 × 10 23
)(
electrons / mol 8.96 × 103 kg / m3 )
= 8.49 × 1028 electrons /m3
−3
M 63.5 × 10 kg / mol
Then from v d = j/ne we have
5.1 × 105 A / m2
vd = 3.8 × 10−5 ms −1 =
= 14 cm / hr
( 8.48 × 10 28
electrons / m 3
)(1.60 × 10 −19
C /electron )
If the electrons drift at such a low speed, why do electrical effects seem to occur
immediately after a switch is thrown, such as when you turn on the room lights? Confusion
on this point results from not distinguishing between the drift speed of the electrons and the
speed at which changes in the electric field configuration travel along wires. This latter speed
approaches that of light. Similarly, when you turn the valve on your garden hose, with the
hose full of water, a pressure wave travels along the hose at the speed of sound in water. The
speed at which the water moves through the hose –– measured perhaps with a dye marker ––
is much lower.

8. A strip of silicon, of cross-sectional width w=3.2 mm and thickness d=250 µm , carries a


current i of 190 mA. The silicon is an n-type semiconductor, having been “doped” with a
controlled amount of phosphorus impurity. The doping has the effect of greatly increasing n,
the number of charge carries (electrons, in this case) per unit volume, as compared with the
21 −3
value for pure silicon. In this case, =
n 8.0 × 10 m . (a) what is the current density in the
strip? (b) what is the drift speed?
i 190 × 10−3 A
Sol. (a)=j = = 2.4 × 105 A /m2
wd ( 3.2 ×10 −3
)(
m 250 × 10 m −6
)
j 2.4 × 105 A / m2
(b) =
vd = = 190 m /s
ne ( 8.0 × 10 21
)(
m−3 1.60 × 10−19 C )
The drift speed (190 m/s) of the electrons in this silicon semiconductor is much
greater than the drift speed ( 3.8 × 10−5 m /s ) of the conduction electrons in the metallic
copper conductor of above problem, even though the current densities are similar. The
number of charge carries in this semiconductor ( 8.0 × 1021 m−3 ) is much smaller than the
CURRENT ELECTRICITY 10

number of charge carries in the copper conductor ( 8.49 × 1028 m−3 ) . The smaller number of
charge carries must drift faster in the semiconductor if they are to establish the same current
density that the greater number of charge carries establish in copper.

9. Find the total linear momentum of the electrons in a conductor of length  = 1000 m carrying
a current i = 70 A .
Sol. Linear momentum of electrons is
P = ( nmA ) vd ………………..(i)
Where e n is the density of electrons, and vd is the drift velocity.
i
vd =
neA
Then linear momentum is
 i  mil
=
P nAm  =  = 0.4 × 10−6 Ns
 neA  e

10. How much time will be taken by an electrons to move a distance  = 1 km in a copper wire of
cross section
A = 1mm2 if it carries a current I = 4.5 A ?
Sol. Time taken by electron to travel in copper wire is
 
t= , Where vd = J or t = where J = i / A
vd ne J / ne
1028 × 1.6 × 10−19 × 103 × 106
Hence, t = neA = = 3 × 106 s
i 4.5

11. Current is flowing from a conductor of non-uniform cross-sectional area. If A1 > A 2 , then find
relation between
a) i1 and i2 b) j1 and j2 c) ( ( vd )1 and ( vd )2 (drift velocity)

where i is current, j is current density, and V is drift velocity.


Sol. a) The charge flowing through a cross section per unit time is i. So, i1 = i2 .
b) Current density is j = i / A. As A1 > A 2 , then j1 < j2 .
c) We know j = nevd , So drift velocity is vd = j/ ne. As j1 < j2 , then ( vd )1 < ( vd ) 2
11 CURRENT ELECTRICITY

INTEXT QUESTIONS

1. How many electrons per second pass through a section of wire carrying a current of 0.7A?
Ans. 0.44 × 1019

2. A current of 7.5 A is maintained in a wire for 45 s. In this time (a) how much charge and (b)
how many electrons flow through the wire?
Ans. 337.5 C, 2.1 × 1021

3. A current of 0.50 ampere is passing through a CuSO4 solution. How many Cu++ ions will be
deposited on cathode in 10 seconds?
25
Ans. × 1019
32

π
4. The current through a wire depends on time as i= i0 + α sin πt, where i0 = 10A and α = A.
2
Find the charge crossed through a section of the wire in 3 seconds, and average current for
that interval.
31
Ans. 31C, A
3

5. Estimate the average drift speed of conduction electrons in a copper wire of cross-sectional
area 1.0 × 10−7 m2 carrying a current of 1.5 A. Assume that each copper atom contributes
roughly one conduction electron. The density of copper is 9.0 × 103 kgm−3 and its atomic mass
is 63.5 amu.
Ans. 1.1mms −1

6. A current of 5 A exists in a 10Ω resistance for 4 minutes.


(i) How many coulombs and
(ii) How many electrons pass through any cross section of the resistor in this time?
Ans. (i) Q = 1200 C n 75 × 1020
(ii) =
A
7. =
The current in a wire varies with time according to the relation I ( 3A ) +  2
t
 S
(a) How may coulomb of charge passes a cross-section of the wire in the time interval
between t = 0 and t = 5s ?
(b) What constant current would transport the same charge in the same time interval?
Ans. a) 40 C b) 10 A

8. A long conductor of charge q, with charge density λ is moving with a velocity 2v parallel to its
own axis. Find the convectional current due to motion of conductor.
Ans. 2λv
CURRENT ELECTRICITY 12

9. If 0.6 mol of electrons flows through a wire in 45 min. what are (a) the total charge that
passes through the wire and (b) the magnitude of the current?
Ans. 5.78 × 104 C , 21.4 A

10. In the Bohr model of the hydrogen atom, an electron in the lowest energy state follows a
circular path 5.29 × 10−11 m from the proton.
(a) show that the speed of the electron is 2.19 × 106 ms −1 .
(b) what is the effective current associated with this orbiting electron?
Ans. b) 1.05 mA

11. The quantity of charge q (in coulombs) that has passed through a surface of area 2cm2 varies
with time according to the equation q = 4t 3 + 5t + 6, where t is in seconds.
(a) what is the instantaneous current through the surface at t = 1s ?
(b) what is the value of the current density?
Ans. a) 17 A b) 85 kAm−2

12. A typical copper wire might have 2 × 1021 free electrons in 1 cm of its length. Suppose that the
drift speed of the electrons along the wire is 0.05 cms −1 . How many electrons would pass
through a given cross section of the wire each second. How large would a current be flowing
in the wire?
Ans. 1020 , 16 A

13. The current in a wire varies with time according to the equation i= 4 + 2t , where i is in
ampere and t is in second. Calculate the quantity of charge that passes through a cross
=
section of the wire during the time =
t 2s to t 6s.
Ans. 48 C

14. An electric current is given by the expression


= I ( t ) 100sin (120πt ) , where I is in ampere and t
 1 
=
is in second. What is the total charge carried by the current from t 0=
to t  s ?
 240 
Ans. 0.265 C

15. A beam contains 2.0 × 108 doubly charged positive ions per cubic centimeter, all of which are
moving toward north with a speed of 1.0 × 105 ms −1. (a) What are the magnitude and direction

of the current density J ? (b) Can you calculate the total current i in this ion beam? If not
what additional information is needed.
Ans. a) 6.4 Am−2 towards north b) No, we need area of cross section of the wire.

16. A very small earthed conducting sphere is at a distance a from a point charge q1 and at a
distance b from a point charge q 2 ( a < b ) . At a certain instant, the sphere starts expanding so
that its radius grows according to the law R = vt. Determine the time dependence I (t) of the
current in the earthing conductor, assuming that the point charges and the centre of the
13 CURRENT ELECTRICITY
sphere are at rest, and in due time the initial point charges get into the expanding sphere
without touching it (through small holes).
  q1 q 2  a
−v  +  t<
  a b  v
 q a b
Ans. 1( t=
)  −v 2 ≤t<
 b v v
 b
 0 t≥
 v

17. (a) The current density in a cylindrical conductor of radius R varies according to the equation
 r
=J J0 1 −  , where r = distance from the axis. Thus the current density is a maximum J0
 R
at the axis r = 0 and decreases linearly to zero at the surface r = R. Calculate the current in
( 2
terms of J0 and the conductor's cross-sectional area A = πR .)
(b) Suppose that instead the current density is a maximum J0 at the surface and decreases
r
linearly to zero at the axis so that J = J0 . Calculate the current.
R
Ans. J
(a) 0 A /3 (b) 2J0 A /3

OHMIC MATERIALS AND OHM’S LAW


Between collisions with the lattice ions, the electrons in a conducting material are
 
accelerated by the electric field E , and so their drift velocity is proportional to E . The
  
current density j is also proportional to v d . So it is reasonable that j should be

proportional to E . In fact, we observe this type of behavior for a wide class of materials. The
proportionality constant between the current density and electric field is the electrical
conductivity σ of the material.
 
j = σE ……………..(i)
A large value of σ indicates that the material is a good conductor of electric current.
The conductivity is a property of the material, not of any particular sample of the material.
The SI unit for conductivity is the siemens per meter (S/m).
Where the siemens is defined as
1 siemens =1 ampere/volt

It is more common to find materials characterized by their resistivity, which is the inverse of
the conductivity.
ρ= 1/ σ ……………….(ii)
in which case Eq. (i) becomes
 
E = ρj ………………(iii)
Units of resistivity are ‘ohm-meter’ where the ohm (symbol Ω ) is defined as
1 ohm = 1 volt/ampere
Note that 1 ohm= (I siemens)–1
CURRENT ELECTRICITY 14

Equation (i) and (iii) are valid only for isotropic materials, whose electrical properties are the
 
same in all directions. In these materials, j will always be in the same direction as E .

We can use Eq. (iii) to determine the resistivity of any material by applying an electric
field and measuring the resulting current density. For some materials, we find that the
resistivity is not a constant but depends on the strength of the electric field. That is, if we
double the electric field the current density does not double. For other materials, we find that
the resistivity does not depend on the strength of the applied field for a wide range of applied
fields. For such materials a plot of E against j gives a straight line, whose slope is the
resistivity ρ . These materials are known as ohmic materials.
Equivalently, such materials are said to satisfy Ohm’s law:
The resistivity (or conductivity) of a material is independent of the magnitude and direction
of the applied electric field).
The above statement can be taken as a statement of ohm’s law.

Many homogeneous materials, including conducting metals such as copper, obey Ohm’s law
for a certain range of values of the applied electric field. If the field is sufficiently large, all
materials will behave in violation of Ohm’s law.

We might also want to know the resistance of a particular object, such as a block of copper of
certain dimensions. Figure below illustrates the situation for a homogeneous, isotropic
conductor of length L and uniform cross-sectional area A, to which we have applied a
potential difference ∆V .

Inside the object, there is a uniform electric field E = ∆V /L . If the current density is also
uniform over the area A, then j = i / A .
The resistivity is then
E ∆V / L
ρ= = ……………………(iv)
j i/A
The quantity ∆V / i that appears in this equation is defined as the resistance R.
∆V
R= ……………………(v)
i
Combining Eq. (iv) and (v) we obtain an expression for the resistance R.
L
R= ρ ………………(vi)
A
15 CURRENT ELECTRICITY
The resistance R is characteristic of a particular object and depends, on the material of
which it is made as well as on its length and cross-sectional area; the resistivity ρ is
characteristic of the material in general. The units of resistance are ohms ( Ω ) .

Equation (v) gives us another basis for stating Ohm’s law. For a particular object, we
can measure the current i for various applied potential difference and plot i as a function of
∆V . If this plot gives a straight line, then the object is ohmic and obeys Ohm’s law. An
equivalent statement of Ohm’s law is:

The resistance of an object is independent of the magnitude or sign of the applied potential
difference.
Ordinary resistors that are found in electric circuits are ohmic for the range of potential
differences that are normally used in circuits. Semiconducting devices, such as diodes and
transistors, usually are nonohmic. Figure below compares the current – voltage plots for
ohmic (figure (a)) and non – ohmic devices (figure (b)).

Note:
1) Keep in mind that the relationship ∆V = iR is not a statement of Ohm’s law. This equation
defines the resistance and is true for both ohmic and nonohmic objects. Even for nonohmic
devices, we can find a value of the resistance R for a particular value of ∆V ; for a different
∆V , a different value of R will be obtained. For ohmic devices, we get the same value of R for
any value of ∆V .

∆V , i, and R are macroscopic quantities, applying to a particular body or extended


 
region. The corresponding microscopic quantities are E , j and ρ ( or σ ) ; they have values at
every point in a body. The macroscopic quantities are related by Eq. (v) ( ∆V =iR ) and the
 
( )
microscopic quantities by Eq. (iii) E = ρ j .

The macroscopic quantities ∆V , i, and R are of primary interest when we are making
electrical measurements on real conducting objects. They are the quantities whose values are
 
indicated on meters. The microscopic quantities E, j and ρ are of primary importance when
we are concerned with the fundamental behavior of matter (rather than of specimens of
matter) as we usually are in the research area of solid state (or condensed matter) physics.
CURRENT ELECTRICITY 16

2) Since,
ρ
R= ………………(i)
A
If a given mass of wire is recast to increase its length (or decrease its radius), then we have
m = volume × density= Ad ………………(ii)
m
⇒ A=
d
ρ ρd 2
Then from
= (i), R =  ……………..(iii)
( m /d ) m
As ρ,d,m are constants
2
2 R 2  1 
⇒ R∝ ⇒ = 
R1   2 
So, we conclude that if a give mass of wire is stretched (or drawn) to n times its original
length, then the new resistance is n2 times original resistance of wire. So,
R new = n2R old
ρm  mρ   1  1
Now,=R =   ⇒ R∝ 2
AdA  A   A 2  A
1
Since A = πr 2 ⇒ R∝
r4

3) Fractional change in resistance:


The resistance of a wire depends on its length L, radius r and cross-sectional area A (where
A = πr 2 ). If the wire is stretched by a small amount, its resistance R changes.
Case-1:
Suppose the length L increases by a small amount ∆L .
Then, we can write
ρL  L2 
R= = ρ  (as AL= volume, V=constant)
A V
Hence fractional change in resistance with a small change in length is ∆R = 2 ∆L
R L
Case-II:
Suppose the wire is stretched so that the cross-sectional area A reduces by small amount ∆A .
Then we can write
ρL  V  ∆R ∆A
R= = ρ 2  ⇒ =
−2
A A  R A
Case-III:
Suppose the wire is stretched so that the radius reduces by ∆r . Then, we can write
ρL  LA  V  ρV  1 ∆R ∆r
R= = ρ 2  = ρ × = 2  4 ⇒ = −4
( πr )  
2
A  A  2 π r R r
All the above cases are valid for small changes only.
OHM’S LAW: A MICROSCOPIC VIEW
17 CURRENT ELECTRICITY
As we discussed previously, Ohm’s law is not a fundamental law of electromagnetism
because it depends on the properties of the conducting medium. The law is very simple in
form, and it is curious that many materials obey it so well, whereas other materials do not
obey it at all. Let us see if we can understand why metals obey Ohm’s law – that is, why their
resistivity ρ are constant (and not, for example, dependent on the applied electric field).

In a metal, the valence electrons are not attached to individual atoms but are free to
move about within the lattice and are called conduction electrons. In copper there is one
such electron per atom, the other 28 remaining bound to the copper nuclei to form ionic
cores.

The theory of electrical conduction in metals is often based on the free-electron model,
in which (as a first approximation) the conduction electrons are assumed to move throughout
the conducting material, somewhat like molecules of gas in a container. In fact, the assembly
of conduction electrons is sometimes called an electrons gas. As we shall see, however, we
cannot neglect the effect of the ion cores on this “gas”.

The classical Maxwellian velocity distribution for the electron gas would suggest that
the conduction electrons have a broad distribution of velocities from zero to infinity, with a
well-defined average. However, in considering the electrons we cannot ignore quantum
mechanics, which gives a very different view. In the quantum distribution, the electrons that
readily contribute to electrical conduction are concentrated in a very narrow interval of
kinetic energies and therefore of speeds. To a very good approximation, we can assume that
the electrons move with a uniform average speed. In the case of copper, this speed is about
v=
av 1.6 × 106 m /s . Furthermore, whereas the Maxwellian average speed depends strongly on
the temperature, the effective speed obtained from the quantum distribution is nearly
independent of temperature.

In the absence of an electric field, the electrons move randomly, again like the
molecules of gas in a container. Occasionally, an electrons collides with an ionic core of the
lattice, suffering a sudden change in direction in the process. As we did in the case of
collision of gas molecules, we can associate a mean free path λ and a mean free time τ to
the average distance and time between collisions. (Collisions between the electrons
themselves are rare and do not affect the electrical properties of the conductor.)

In an ideal metallic crystal (containing no defects or impurities) at 0 K, electron-lattice


collisions would not occur, according to the predictions of quantum physics; that is, λ → ∞
as T → 0 K for ideal crystals. Collisions take place in actual crystals because (1) the ionic
CURRENT ELECTRICITY 18

cores at any temperature T are vibrating about their equilibrium positions in a random way;
(2) impurities –– that is, foreign atoms –– may be present; and (3) the crystal may contain
lattice imperfections, such as missing atoms and displaced atoms. Consequently, the
resistivity of a metal can be increased by (1) raising its temperature, (2) adding small
amounts of impurities, and (3) straining it severely, as by drawing it through a die, to
increase the number of lattice imperfections.

When we apply an electric field to a metal, the electrons modify their random motion
in such a way that they drift slowly, in the opposite direction to that of the field, with an
average drift speed v d . This drift speed is very much less (by a factor of something like1010 )

than the effective average speed v av . Figure below suggests the relationship between these
two speeds.

The solid lines suggest a possible random path followed by an electron in the absence
of an applied field; the electrons proceeds from x to y, making six collisions on the way. The

dashed line show how this same event might have occurred if an electric field E had been
applied. Note that the electron drifts steadily to the right, ending at y ' rather than at y. In
preparing the above figure, it has been assumed that the drift speed v d is 0.02 v av ; actually,
it is more like 10−10 v av , so that the “drift” exhibited in the figure is greatly exaggerated.

We can calculate the drift speed v d in terms of the applied electric field E and of v av
and λ . When a field is applied to an electron in the metal, it experiences a force eE,
which imparts to it an acceleration a given by Newton’s second law.
eE
a=
m
Consider an electron that has just collided with an ion core. The collision, in general,
momentarily destroys the tendency to drift, and the electron has a truly random direction
after the collision. During the time interval to the next collision, the electron’s speed changes,
on the average, by an amount a ( λ / v av ) or aτ , where τ is the mean time between collisions.
We identity this with the drift speed v d , or
eEτ
v d = aτ = ……………………(i)
m
19 CURRENT ELECTRICITY

j eEτ
We may also express ρ in terms of the current density, which gives =
vd =
ne m
Combining this with ρ =E / j , we finally obtain
m
ρ= …………………….(ii)
ne2 τ

Note that m, n, and e in equation (ii) are constants. Thus Eq. (ii) can be taken as a
statement that metals obey Ohm’s law if we can show that τ is a constant. In particular, we
must show that τ does not depend on the applied electric field E. In this case ρ does not
depends on E, which is the criterion that a material obey Ohm’s law. The quantity τ depends
on the speed distribution of the conduction electrons. We have seen that this distribution is
affected only very slightly by the application of even a relatively large electric field, since v av is
of the order of 106 m /s , and v d is only the order of 10−4 m /s , a ratio of 1010 . Whatever the
value of τ is (for copper at 20°C , say) in the absence of a field, it remains essentially
unchanged when the field is applied. Thus the right side of Eq. (ii) is independent of E( which
means that ρ is independent of E), and the material obeys Ohm’s law.

1
Note: It may be tempting to write Eq. (i) as v=
d aτ , reasoning that aτ is the electron’s final
2
1
velocity, and thus its average velocity is half that value. The extra factor of would be
2
correct if we followed a typical electron, taking its drift speed to be the average of its velocity
over its mean time τ between collisions. However, the drift speed is proportional to the
current density j and must be calculated from the average velocity of all the electrons taken
at one instant of time. For each electron, the velocity at any time is at, where t is the time
since the last collision for that electron. Since the acceleration a is the same for all electrons,
the average value of ‘at’ at a given instant is aτ , where τ is the average time since the last
collision, which is the same as the mean time between collisions.

Mobility
Conductivity arises from mobile charge carriers. These mobile charge carriers are
electrons in case of metal, electrons and positive ions in case of an ionized gas and both
positive and negative ions in case of an electrolyte. In semiconductor’s material such as
germanium and silicon, conduction is partly due to electrons and partly due to electron
vacancies called holes. Holes are sites of missing electrons which acts like positive charges.
The mobility µ of a charge carrier is defined as the drift velocity per unit electric field.
vd vd qτE   qE  
=
µ = , also vd =  vd = aτ =  m  τ 
E E m    
vd qτ
⇒ =
µ =
E m
CURRENT ELECTRICITY 20

LEARN THE SKILLS


12. (a) What is the mean free time τ between collision for the conduction electrons in copper? (b)
What is the mean free path λ for these collisions? Assume an effective speed v av of
1.6 × 106 m /s . ( ρcopper = 1.69 × 10−8 Ω m and n = 8.49 × 1028 m−3 ).
m 9.11 × 10−31 kg
Sol. (a) τ = =
ne ρ ( 8.49 × 1028 m−3 )(1.60 × 10−19 C )2 (1.69 × 10−8 Ω .m )
2

(b) We define the mean free path from


( )( )
λ = τv av = 2.48 × 10−14 s 1.6 × 106 m /s =
4.0 × 10 m =−8
40 nm
This is about 150 times the distance between nearest-neighbor ions in a copper lattice. A full
treatment based on quantum physics reveals that we cannot view a “collision” as a direct
interaction between an electron and an ion. Rather, it is an interaction between an electrons
and the thermal vibrations of the lattice, lattice imperfections, or lattice impurity atoms. An
electron can pass very freely through an “ideal” lattice –––– that is, a geometrically “perfect”
lattice close to the absolute zero of temperature. Mean free paths as large as 10 cm have
been observed under such conditions.

13. The dimensions of a conductor of specific resistance ρ are shown below.

Find the resistance of the conductor across AB, CD and EF.



Sol. R = ρ , Resistance across AB, CD and EF in tabular form is shown below.
A
 A R
c
AB c a×b ρ
ab
b
CD b a ×c ρ
ac
a
EF a b×c ρ
bc

14. If a wire is stretched to double its length, find the new resistance if the original resistance of
the wire was R.
Sol. We know that , R = ρ
A
ρ '
Let new resistance be, R ' =
A'
ρ × 2
A '  ' = A ;if  ' = 2 then A ' = A /2 . Thus
= R ' = 4 ρ = 4R
A /2 A
21 CURRENT ELECTRICITY
15. The wire is stretched to increase the length by 1%. Find the percentage change in the
resistance.
As we know that, for constant volume R ∝  , so, ∆R % =2∆
2
Sol. %=2 × 1% =2%
R 
As percentage change is positive, hence resistance will increases by 2%.

16. A 3000 km long cable consists of seven copper wires, each of diameter 0.73 mm, bundled
together and surrounded by an insulating sheath. Calculate the resistance of the cable. Use
3 × 10−6 Ω cm for the resistivity of the copper.
ρ
Sol. The resistance R of a conductor is related to the resistivity ρ by R =
, where  and A are
A
the length of the conductor and the cross-sectional area, respectively. Since the cable
consists of N=7 copper wires, the total cross sectional area is
2
πd2 π ( 0.073 cm )
A =Nπr2 =N =7
4 4
The resistance then becomes
ρ
R= =
( )(
3 × 10−6 Ωm 3 × 108 m )
=3.1 × 104 Ω ⇒ R = 31 kΩ
A 2
7π ( 0.073 cm )
4

17. The voltage-current graphs for two resistors of the same material and the same radius with
length, L1 and L 2 are shown in figure. If L1 > L 2 , state with reason which of these graphs
represents voltage-current change for L1 .

=
Sol. As R A V=
/IA and R B V /IB
IA R B
=
IB R A
From the figure, as IA > IB , R B > R A . Since, R ∝ L , graph B represents the voltage-current
change for L1 .

18. You need to produce a set of cylindrical copper wires 2.5 m long that will have a resistance of
0.125 Ω each. What will be the mass of each of these wires? (Density of copper is
8.9 × 103 kgm−3 and resistivity of copper is 1.72 × 10−8 Ω m ).
Sol. =
Given that L=2.5 m and R 0.125 Ω . To find the mass of each wire, we need to calculate the
volume of one of the wires.
=Mass Density × Volume
CURRENT ELECTRICITY 22

 ρL ρL 
Volume =
Area × Length  R == or A
 A R 
−8 2
ρL2 1.72 × 10 × ( 2.5 )
= AL
Volume = = = 8.6 × 10−7 m3
R 0.125
m = d × V = 8.9 × 103 × 8.6 × 10−7 =7.654 × 10−3 kg =7.654 g

19. The resistivity of sea water is about 25 Ω cm . The charge carries are chiefly Na + and C − ions,
and of each there are about 3 × 1020 cm−3 . If we fill a plastic tube 2 m long with sea water
and connect a 12 V battery to the electrodes at each end, what is the resulting average drift
velocity of the ions, in cms −1 ?
Sol. The current in a conductor of cross sectional area A is related to the drift speed vd of the
charge carriers by
I = neAvd
Where n is the number of charges per unit volume. We can then rewrite the Ohm’s Law as
ρ 
= =
V IR ( neAvd ) =  nevd ( ρ )
A
V
⇒ vd =
neρ
12 V
Substituting the values, we get vd =
(6 ×10
20
cm −3
)(1.6 ×10−19 C ) (25 Ω cm)(200 cm)
Vcm
⇒ 2.5 × 10−5
vd = 2.5 × 10−5 cms −1
=
CΩ

20. A wire of mass m, length  , density d, and area of cross section A is stretched in such a way
that its length increases by 10% of its original value. Express the changed resistance in
percentage.
Sol. Given mass m, length 1 =  , density d, and area of across section A1 = A . Let ρ be the
resistivity and R1 be the resistance of the wire.
Mass of wire m = volume × density = A × d = Ad . Therefore, area of cross section is
A1 = m / d , and the resistance of the wire is
 2d  ρd  2
R1 =
ρ =
ρ =  =k 2 …………….(i)
A1 m m
10
Let  2 be the new length, then, 2 =
+ =  + 0.1 =
1.1
100
Let R 2 be the resistance of the wire after stretching, then R 2 = k22 …………(ii)
R 2 22
Dividing Eq. (ii) by Eq. (i), we have = =
(1.1)2 =
2
1
1.21
R1 12 12
or R=
2 1.21 R=
1 R1 + 0.21R1 or R 2 − R1 =
0.21 R1
R 2 − R1
Hence, the percentage change in the resistance is × 100 =
21%
R1
23 CURRENT ELECTRICITY

21. Consider a wire of length 0.1 m with an area of cross section 1mm2 connected to 5 V. Find
5 × 10−6 m2 V −1s −1
the current flowing through the metallic wire where µ is mobility =
1.6 × 10−19 C, and n =
e= 8 × 1028 m−3
Sol. =
Given =
A 1mm2
10−6 m
=2
=
,  0.1m, V 5V
Due to the applied potential difference across the wire, an electric field is set up in the
conductor. So
V 5
=
E = = 50 Vm−1
 0.1
The current flowing through the wire is given by
= I n Aev= d n AeµE [where µ =vd /E ]
8 × 1028 × 10−6 × 1.6 × 10−19 × 5 × 10−6 × 50 =
= 3.2 A

22. A rod of length L and cross-section area A lies along the x-axis between x=0 and x=L. The
material obeys Ohm’s law and its resistivity varies along the rod according to.
ρ0e − x/L
ρ(x) =

The end of the rod at x=0 is at a potential V0 and it is zero at x=L.


a) Find the total resistance of the rod and the current in the wire
b) Find the electric potential V(x) in the rod as a function of x.
Sol. a) Resistance of elementary section dx at x=x is,
ρ ( x ) dx  
dR = R = ρ 
A  A
x

ρ0 L
e .dx
=
A
Since all such elements are in series.
x
Therefore,
= R
L
∫=
0
dR
ρ0
A
L −
∫0 e=
ρ0L
L dx
A
( )
1 − e −1 or=R
ρ0L 
A 

1
1− 
e
V0 V0 A  e 
Current in the wire, =i =  
R ρ0L  e − 1 
b) ( dV ) = idR
V0 A  e  ρ0e − x/L
dV =   .dx
ρ0L  e − 1  A
V0  e  − x/L
( dV ) =  e dx
L  e −1


V V0  e  x − x/L
∫V0 dV =
L  e − 1  ∫0
  e dx or =
 e 
V − V0 V0  (
 1− e
1 − e 
− x/L
)

(
V0 e − x/L − e −1
V = −1
)
1− e
CURRENT ELECTRICITY 24

23. Consider a material of resistivity ρ in a shape of a truncated cone of altitude h, and radii a
and b, for the right and the left ends, respectively, as shown in the figure.

Assuming that the current is distributed uniformly throughout the cross-section of the cone,
what is the resistance between the two ends?
Sol. Consider a thin disk of radius r at a distance x from the left end. From the figure shown, we
b−r b−a x
have = ⇒ r =( a − b ) + b
x h h

ρ
Since resistance R is related to resistivity ρ by R =
, where  is the length of the conductor
A
and A is the cross section, the contribution to the resistance from the disk having a
thickness dy is
ρ dx ρdx
=dR = 2 2
πr 
π b +
( a − b) x 

 h 
h
ρdx ρh ρh
=
Straight forward integration then yields R ∫=2 πab
⇒ R=
πab
0 
π b +
(a − b) x 

 h 
du 1
Where we have used, ∫ ( αu + β )2 = −
α ( αu + β )
ρh ρ
Note that if b=a, we get=
R = 2 A
πa

24. Consider a hollow cylinder of length L and inner radius a and outer radius b, as shown in
figure. The material has resistivity ρ /
25 CURRENT ELECTRICITY

a) Suppose a potential difference is applied between the ends of the cylinder and produces a
current flowing parallel to the axis. What is the resistance measured?
b) If instead the potential difference is applied between the inner and outer surfaces so that
current flows radially outward, what is the resistance measured?
Sol. a) When a potential difference is applied between the ends of the cylinder, current flows

( )
π b2 − a 2 , and the resistance
parallel to the axis. In this case, the cross-sectional area is A =

ρL ρL
is given by =
R =
A (
π b − a2
2
)
b) Consider a differential element which is made up of a thin cylinder of inner radius r and
the outer radius r + dr and length L. Its contribution to the resistance of the system is given
by
ρd ρdr
=dR =
A 2πrL
Where A= 2πrL is the area normal to the direction of current flow. The total resistance of the
b
ρdr ρ b
system becomes
= R ∫=
2πrL 2πL
log e  
a
a

25. Show that the total amount of charge at the junction of the two materials in figure is
 1 1
ε0 I  −  , where I is the current flowing through the junction, and σ1 and σ2 are the
 σ2 σ1 
conductivities for the two materials.


Sol. In a steady state of current flow, the normal component of the current density j must be the
same on both sides of the junction. Since j = σE , we have σ1E1 =
σ2E2
 σ1 
⇒ E2 =
  E1
 σ2 
CURRENT ELECTRICITY 26

Let the charge on the interface be q enc , then from the Gauss’s Law, we have
  q enc q enc
∫ E.dA =( E2 − E1 )A = ε0 ⇒ E2 − E1 =
Aε 0
Substituting the expression for E2 from above then
σ   1 1
ε0 A E1  1 − 1 =
q enc = ε0 Aσ1E1  − 
 σ2   σ2 σ1 
Since the current is I = JA = ( σ1E1 ) A . So, the amount of charge on the interface becomes.
 1 1
q enc =
ε0 I  − 
 σ2 σ1 

INTEXT QUESTIONS
18. Consider a wire of length  , area of cross section A, and resistivity ρ with resistance 10 Ω .
Its length is increased by applying a force, and it becomes four times of its original value.
Find the changed resistance of the wire.
Ans. 160 Ω

19. A steady current flows in a metallic conductor of non-uniform cross section. State which of
the quantities i.e., current, current density, electric field, and drift velocity remain constant?
Ans. Current is constant while current density, electric field, and drift velocity are not constant
and all vary inversely with area of cross section.

20. A cylindrical conducting wire of radius 0.2 mm is carrying a current of 20 mA . (a) How many
electrons are transferred per second between the supply and the wire at one end? (b) Write
down the current density in the wire.
1
Ans. (a) 1.25 × 1017 (b) × 106 A / m2 .

21. A battery sets up an electric field of 25 N/C inside a uniform wire of length 2 m and a
resistance of 5Ω . Find current through the wire.
Ans. 10A.

22. A uniform copper wire of mass 2.23 × 10−3 kg carries a current of 1 A when 1.7 V is applied
across it. Calculate the length and the area of cross section. If the wire is uniformly stretched
to double its length, calculate the new resistance. Density of copper is 8.92 × 103 kgm−3 and
resistivity is 1.7 × 10−8 Ω m .
Ans. 5 m, 5 × 10−8 m2 , 6.8 Ω

23. If a copper wire is stretched to make it 0.1% longer, what is the percentage change in its
resistance?
Ans. 0.2%

24. A rectangular carbon block has dimensions 1.0cm × 1.0cm × 50cm.


27 CURRENT ELECTRICITY
(i) What is the resistance measured between the two square ends?
(ii) Between two opposing rectangular faces? Resistivity of carbon at 20°C is 3.5 × 10−5 Ωm.
Ans. (i) 0.175Ω (ii) 7 × 10−5 Ω

25. A piece of wire of uniform cross section has a resistance 8 Ω . If the length of the wire is
doubled and its area of cross section is increased four times, calculate the new resistance.
Neglect the temperature variation of resistance.
Ans. 4Ω

26. A steady current passes through a cylindrical conductor. Is there an electric field inside the
conductor?
Ans. Yes

27. A potential difference V is applied to copper wire of diameter d and length L. What will be the
effect on the electron drift speed by doubling (a) voltage V (b) length L (c) diameter d?
Ans. a) Doubled b) halved c) will not change

28. A wire has a resistance R. What will be its resistance if


(a) it is double on itself and (b) it is stretched so that
(i) its length is doubled and (ii) its radius is halved.
R
Ans. a) b) i) 4R ii) 16R
4

29. The following table gives the length of three copper rods, their diameters, and the potential
differences between their ends. Rank the rods according to (a) the magnitude of the electric
field within them, (b) the current density within them, and (c) the drift speed of electrons
through them, greatest first.
Rod Length Diameter P.D.
1 L 3d V
2 2L d 2V
3 3L 2d 2V
Ans. I1 > I3 > I2

30. The voltage –current variation of two metallic wires X and Y at constant temperature is
shown in Figure. The wires have the same length and the same diameter. Which of the two
wires will have larger resistivity.

Ans. ρ Y > ρX
31. 1 m long metallic wire is broken into two unequal parts P and Q. P of the wire is uniformly
extended into another wire R. Length of R is twice the length of P and the resistance of R is
CURRENT ELECTRICITY 28

equal to that of Q. Find the ratio of the resistance of P and R and also the ratio of lengths of P
and Q.
1 1
Ans. ,
4 4

32. A uniform copper wire of mass 2.23 × 10−3 kg carries a current of 1 A when 1.7 V is applied
across it. Calculate its length and area of cross section. If the wire is uniformly stretched to
double its length, calculate the new resistance. Density of copper is 8.92 × 103 kgm−3 and
resistivity is 1.7 × 10−8 Ωm
Ans. 6.8 Ω

33. Material with uniform resistivity ρ is formed into a wedge as shown in figure. Show that the
resistance between face A and face B of this wedge is

L y 
R= ρ og e  2 
w ( y 2 − y1 )  y1 

34. The region between two concentric conducting spheres of radii ra and rb is filled with a
conducting material of resistivity ρ .
1 1 ρ
(a) Show that the resistance between the spheres is given=
by, R  − .
 ra 4π
rb 
(b) Derive an expression for the current density as a function of radius, if the potential
difference between the spheres is Vab .
(c) Find the electric field at distance 'r' from the centre in part (b)
 V rr   V rr 
Ans. (b)  2 ab a b  (c)  2 ab a b 
 ρr (rb − ra )   r (rb − ra ) 
29 CURRENT ELECTRICITY

TEMPERATURE VARIATION OF RESISTIVITY


Figure shows a summary of some experimental measurements of the resistivity of
copper at different temperatures. For practical use of this information, it would be helpful to
express it in the form of an equation. Over a limited range of temperature, the relationship
between resistivity and temperature is nearly linear. We can fit a straight line to any selected
region of the figure, using two points to determine the slope of the line. Choosing a reference
points, such as that labeled T0 , ρ0 in the figure, we can express the resistivity ρ at an
arbitrary temperature T from the empirical equation of the straight line in the figure, which is
ρ − ρ0 = ρ0 α av ( T − T0 ) …………………(i)

(This expression is very similar to that for linear thermal expansion. ∆L =αL ∆T . We have
written the slope of this line as ρ0 α av . If we solve Eq. (i) for α av , we obtain
1 ρ − ρ0
α av = ………………… (ii)
ρ0 T − T0
The quantity α av is the mean (or average) temperature coefficient of resistivity over the region
of temperature between the two points used to determine the slope of the line. We can define
a more general temperature coefficient of resistivity as
1 dρ
α= ………………… (iii)
ρ dT
Which is the fractional change in resistivity dρ / ρ per change in temperature dT. That is, α
gives the dependence of resistivity on temperature at a particular temperature, whereas α av
gives the average dependence over a particular interval. The coefficient α is in general
dependent on temperature.

For most practical purpose, Eq. (i) gives results that are within the acceptable range of
accuracy. For more precise work, such as the use of the platinum resistance thermometer to
measure temperature, the linear approximation is not sufficient. In this case we can add
( T − T0 ) and ( T − T0 ) to the right side of Eq. (i) to improve the precision. The
2 3
terms in
CURRENT ELECTRICITY 30

coefficients of these additional terms must be determined empirically, in analogy with the
coefficient α av in Eq. (i).

The resistance of a given conductor depends on its length and area of cross-section
besides the resistivity. As temperature changes, the length and the area also change. But
these changes are quite small and the factor  / A may be treated as constant. Then R ∝ ρ
and hence
R ( T ) R ( T0 ) 1 + α [ T − T0 ]
=

LEARN THE SKILLS


26. A copper coil has a resistance of 20.0 Ω at 0 °C and a resistance of 26.4 Ω at 80 °C . Find the
temperature coefficient of resistance of copper.
Sol. R=80 °C R 0 °C [1 + α ∆T ]
26.4
= 20.0 1 + α × ( 80 − 0 ) 
or 26.4 or =+
1 80 α
20
On solving, we get α = 4 × 10−3 °C−1

27. A metallic wire has a resistance of 120 Ω at 20 °C . Find the temperature at which the
resistance of same metallic wire rises to 240 Ω where the temperature coefficient of the wire
is 2 × 10−4 C −1 .
−4
Given=
Sol. R1 R= 20 120 Ω , R =2 R =
T 240 Ω , α = 2 × 10 °C−1 , =t1 20 °C, t2= T= ?
Formula used to find the temperature coefficient of resistance is
R 2 − R1 R T − R 20 240 − 120
=α = or 2 × 10−4 =
R1 ( t2 − t1 ) R 20 ( T − 20 ) 120 ( T − 20 )
120 1
or T − 20 = or T= + 20 = 0.5 × 104 + 20 = 5020 °C
120 × 2 × 10−4 2 × 10−4

28. The I-V characteristics of a resistor is observed to deviate from a straight line for higher value
of current as shown in figure. Why?

Sol. For higher value of current, the resistor gets heated and consequently its resistance
increases. The resistor becomes non-ohmic due to which the I-V characteristic deviates from
straight line thereby showing lesser current for the same voltage.

29. Two resistors with temperature coefficients of resistance α1 and α2 have resistance
R 01 and R 02 at 0 °C . Find the temperature coefficient of the compound resistor consisting
of the two resistors connected.
31 CURRENT ELECTRICITY
a) in series b) in parallel
Sol. In series:

At 0 °C R 01 R 02 =
R 0 R 01 + R 02
At t °C R 01 (1 + α1t ) R 02 (1 + α2t ) R 0 (1 + αt )
R 01 (1 + α1t ) + R 02 (1 + α=
2t ) R 0 (1 + αt )
or R 01 (1 + α1t ) + R 02 (1 + α=
2t ) ( R 01 + R 02 ) (1 + αt )
R 01α1 + R 02α2
∴ R 01 + R 01α1t + R 02 + R 02α2t = R 01 + R 02 + ( R 01 + R 02 ) αt or α= ’
R 01 + R 02
In parallel:

1 1 1
At t °C= +
R 0 (1 + αt ) R 01 (1 + α1t ) R 02 (1 + α2t )
R 01 + R 02 1 1
or = +
R 01R 02 (1 + αt ) R 01 (1 + α1t ) R 02 (1 + α2t )
Using the Binomial expansion, we have
1 1 1 1
(1 − αt ) + (1=
− αt ) (1 − α1t ) + (1 − α2t )
R 02 R 01 R 01 R 02
 1 1  α1 α α1R 02 + α2R 01
i.e., αt  + = t+ 2 t or α=
 R 01 R 02  R 01 R 02 R 01 + R 02

1 dρ
30. The temperature coefficient of resistivity α is given by α = , where ρ is the resistivity at
ρ dT
temperature T.
a
a) Assume that α is not constant and is given by α = , where T is the absolute temperature
T
b
where a is a constant, show that the resistivity ρ is given by ρ = , where b is another
T
constant.
−1
b) Using the values =ρ 3.5 × 10−5 Ωm and α = −5 × 10−4 ( C° ) for graphite at 293 K, determine
a and b.
c) Using your result from part (b), determine the resistivity of graphite at −196 °C and
300 °C . (Remember to express T on absolute scale).
1 dρ dρ dT
Sol. a) α = ⇒ =α dT = −a
ρ dT ρ T
ρ T
dρ dT
Let ρ = ρ0 at T = T0 , then ∫ = −a ∫
ρ T
ρ0 T0
CURRENT ELECTRICITY 32

a
 ρ   T   T0 
⇒ log e  
 ρ0 
=
−a log e 
 T0 
=
log e
 T 
 ⇒ ρ= (ρ0 T0a ) T1a = b
Ta
Here b = ρ0 T0a
−4 −1
b) Given that ρ0= 3.5 × 10−5 Ω m and α0 =−5 × 10 ( °C ) at T0 = 293 K
 a
a = −α0 T0 as α = − 
 T
⇒ ρ0 T0a =
a ≈ 0.15 ’ and b = ( )
3.5 × 10−5 ( 293 )
0.15
8 × 10−5 Ω mK
=
[be careful while taking units of a and b]
b 8 × 10−5
c) Now at T =
−196 + 273 =
77 K, =
ρ = ≈ 4.3 × 10−5 Ω m
Ta 0.15
( 77 )
8 × 10−5
Similarly at T = 300 + 273 = 573
= K, ρ
0.15
≈ 3 × 10−5 Ω m
( 573 )
INTEXT QUESTIONS
35. A coil of wire has a resistance of 25.00 Ω at 20° C and a resistance of 25.17 Ω at 35° C .
What is its temperature coefficient of resistance?
Ans. 4.5 × 10−4 C −1

36. A metal wire of diameter 2 mm and of length 300 m has a resistance of 1.6424 Ω at 20° C
and 2.415 Ω at 150° C . Find the values of α, R 0 , ρ0 and ρ20 °C where the zero subscript refers
to 0°C
Ans. 3.9 × 10−3C −1 , 1.5236 Ω ,1.596 × 10−8 Ω − m , 1.720 × 10−8 Ω − m

37. It is desired to make a 20 Ω coil of wire, which has a zero thermal coefficient of resistance. To
do this, a carbon resistor of resistance R1 is placed in series with an iron resistor of
resistance R 2 . The proportions of iron and carbon are so chosen that R1 + R 2 = 20 Ω for all
temperatures near 20 °C . Find R1 and R 2 ? α carbon =
−0.5 × 10−3 °C −1, α iron =
5 × 10−3 °C −1
Ans. 18.18 Ω , 1.82 Ω

38. A resistance R of thermal coefficient of resistivity α is connected in parallel with a resistance


3R, having thermal coefficient of resistivity 2 α . Find the value of αeff .
5
Ans. α
4

39. A potential difference of 200 volt is applied to a coil at a temperature of 15°C and the current
is 10 A. What will be the temperature of the coil when the current has fallen to 9 A, the
applied voltage being the same as before? Temperature coefficient of resistance
1
( α=) °C −1.
234
Ans. 41°C
33 CURRENT ELECTRICITY

40. The V-I graph for a metallic wire at two different temperatures T1 and T2 is shown. Which of
the two temperatures is higher?

Ans. T2 > T1

41. A copper wire having cross-sectional area of 0.5 mm2 and a length of 0.1 m is initially at
25 °C and is thermally insulated from the surrounding. If a current of 10 A is set up in this
wire, (a) find the time in which the wire will start melting. The change of resistance with the
temperature of the wire may be neglected. (b) What will this time be, if the length of the wire
is doubled? For copper, resistivity is 1.6 × 10−8 Ωm , density is 9,000 kgm−3 , and specific heat
capacity is 0.09ca g −10 C −1. Melting temperature of copper is 1075 °C .
Ans. a) 9 min 16 s, b) 9 min 16 s

42. (a) At what temperature would the resistance of a copper conductor be double of its value of
0°C ? (b) Does this same temperature hold for all copper conductors, regardless of shape and
size?  α C= 4.0 × 10−3 °C −1 
Ans. a) 250 °C b) YES

43. A potential difference is applied across the filament of a bulb at t = 0, and it is maintained at
a constant value while the filament gets heated to its equilibrium temperature. We find that
the final current in the filament is one-sixth of the current drawn at t = 0 . If the temperature
of the filament at t = 0 is 20 °C and the temperature coefficient of resistivity at 20 °C is
0.0043 °C −1 , find the final temperature of the filament.
Ans. 1182.8 °C

44. When a metal rod is heated, not only its resistance but also its length and area of cross-
section changes. Find the percent change in R,  and A of a copper wire for a temperature
rise of 1°C. Coefficient of linear expansion for copper is 1.7 × 10−5 ( °C ) and its thermal
−1

coefficient of resistance is 3.9 × 10−3 ( °C ) .


−1

Ans. 0.39%, 0.0017%, 0.0034% respectively

45. Two coils connected in series have resistance of 600 Ω and 300 Ω and temperature co-
0.001( °C ) and 0.004 ( °C ) respectively at
−1 −1
efficient of 20 °C . Find resistance of the
combination at a temperature of 50 °C . What is the effective temperature co-efficient of
combination.
Ans. 954 Ω , 0.002 ( °C )−1
CURRENT ELECTRICITY 34

ELECTRIC CIRCUITS
Introduction
An electric circuit is any arrangement of conductors in which current can flow around
one or more closed conducting paths. Since in most circumstances the flow of current
involves energy losses, a necessary component of any practical circuit is a source of energy,
called an electromotive force, abbreviated emf. The energy to move current around the circuit
cannot come from an electrostatic field, since, as we have seen, an electrostatic field cannot
do net work on charge taken around a closed path back to where it started. Thus an emf
must be a non-electrostatic source of energy.

The part of the circuit to which energy is delivered is called the load. In it the electric
energy is transformed into heat in a resistor or mechanical energy in a motor, or light in an
electric lamp, or chemical energy in an electroplating bath. Conducting wires provide the
path for current to flow from the source of emf to the load and back to the source of emf.
Usually, the loss of energy in the wires connecting source and load is minor comparted with
the energy transformation in the load.

In this chapter our concern is with circuits in which the current flow is steady. A
steady-state circuit must provide a complete path which allows charge to move continuously,
first through the energy source, where its potential energy is raised, then through the load
elements in which its potential energy is transformed, and finally back to its starting point, to
repeat the process.

Basic Electric Circuit


Figure (1) illustrates general problem we analyze in this chapter. A battery is
connected to a “device”. The device may be a single circuit element, such as a resistor or a
capacitor, or it may be a combination of circuit elements. The battery maintains the upper
terminal at a potential V and the lower terminal at a potential V . For an ideal battery, the
potential difference V  V between its terminals is independent of the amount of current
that it is providing to the circuit. As we discuss later in this chapter, for real batteries the
potential difference does depend on the current.
35 CURRENT ELECTRICITY

In the electrostatic case, in which conductors are equipotentials, the potential V at


the positive terminal of the battery would characterize the entire wire connecting the upper
end of the device to the battery. In this case, the potential difference V  V between the
battery terminals would also appear between the upper and lower terminals of the device.
When currents are flowing in the wires, the conclusions of electrostatics are no longer valid.
When a current i flows in a conductor there is a potential difference V  iR across the
conductor. However, the resistance of the wires is usually very small compared with the
resistance of the device in our circuit, so we are usually justified in neglecting the effect of
the wires; in particular, we assume that there is no potential drop in the wires, and in this
case the full voltage difference of the battery terminals does appear across the terminals of
the device.

The battery can be considered a “pump” for charge, as if it were bringing positive
charge through the battery from the negative terminal to the positive one. In actuality, it is
usually the motion of the negatively charged electrons that is responsible for the current
flow. Another way of interpreting the flow of charge in the circuit is to regard the positive
charge as “falling” through the device from a region of high potential (the part of the device
connected to the positive terminal of the battery) to a region of lower potential (the part of the
device connected to the negative terminal of the battery).

The function of the battery in the circuit is to maintain the potential difference that
enables the flow of charge. The battery is not a source of electrons. Electrons pass through
the battery and have their energy raised as they move inside the battery from the positive to
the negative terminal. When we say that a battery is “drained,” we do not mean that it has
“used up” its supply of electrons; instead, we mean that we have exhausted the source of
energy (often a chemical reaction) that was responsible for raising the energies of the
electrons. Note in figure (1) that the electrons move through out the entire circuit; they do not
“come from” the battery.

When we first connect the battery to the device, the circuit behaves in an irregular
manner. The situation is similar to what occurs when you first turn on a garden hose
connected to a sprinkler. At first the water gushes through the hose, creating whirlpools and
eddies. When it reaches the sprinkler it may at first by chance emerge from some of the holes
and not from others. After a few seconds a steady flow is established, and the water flows
past any point at a constant rate. In electrical circuits, we usually ignore this initial behavior
(called the transient behavior) and consider only the steady situation, which is reached very
rapidly (within nanoseconds).
The symbols of basic circuit elements of this chapter are shown in below.
CURRENT ELECTRICITY 36

Electromotive Force
Every complete circuit in which there is a steady current there must be some portion
of the loop where a charge travels from lower to higher potential, despite the electrostatic
force trying to push it from higher to lower potential. The influence that makes charge move
from lower to higher potential is called electromotive force.

Batteries, generators, photovoltaic cells, and thermocouples are examples of such


devices, which are called seats of electromotive force. Any such device can transfer energy
into a circuit in which it is connected; thus such a device is sometimes called a source,
although the term energy converter is preferable. Electromotive force is usually abbreviated
emf, pronounced “ee-em-eff.”

Figure below is a schematic representation of a seat of emf, such as a battery or


generator. Such a device has the property that it can maintain a potential difference between
conductors a and b, called the terminals of the device. In the figure there is no conducting
path outside the device connecting a and b, and the device is said to be on open circuit.

Terminal a, marked +, is maintained by the sources at a higher potential than


terminal b, marked –. Associated with this potential difference is an electrostatic field Ee , at
all points between and around the terminals, both inside and outside the source. The
electrostatic field Ee inside the device is directed from a toward b, as shown. The source is
itself a conductor, however, and if the only force on the free charges within it were that
exerted by the electrostatic field, the positive charges would move from a towards b (or the
negative charges from b toward a). The excess charges on the terminals would decrease, and
the potential difference between them would decrease and eventually become zero.

But this is not the way batteries and generators actually act; in fact, they maintain a
potential difference even when there is a steady current. From this we conclude that there
must be some additional force on the charges within the source, tending to push them from a
lower-potential point to a higher, opposite to the tendency of the electrostatic force. The
origin of the non-electrostatic force depends on the nature of the source. In a generator it
results from the action of a magnetic field on moving charges. In a battery it is associated
with varying electrolyte concentrations arising from chemical reactions. In an electrostatic
machine such as a Van de Graaff or Wimshurst generator, it is a mechanical force applied by
a moving belt or wheel.

Whatever the origin of the non-electrostatic force, which we may call F n , its effect is
the same as though there were an additional electric field En , of non- electrostatic origin,
related to the force by F n  qEn . That is, the non-electrostatic force is the same as if there
were a non-electrostatic field En in addition to the purely electrostatic field Ee ,
37 CURRENT ELECTRICITY
When the source is on open circuit, as in the above figure, the charges are in
equilibrium, and the resultant field E , the vector sum of Ee and En , must be zero at every
point.
Ee  En  0

Now the electrostatic potential difference Vab is defined as the work per unit charge
preformed by the electrostatic field Ee on a charge moving from a to b. Similarly, we may
consider the work done by the non-electrostatic field En . It is customary to speak of the
(positive) work of this field during a displacement from b to a, rather than the reverse.
Specifically, the work performed by En , per unit charge, when a charge moves from b to a, is called
the electromotive force  of the source.

When Ee  En , we have Vab   . Hence for a source on open circuit, the potential
difference Vab or the open circuit terminal voltage is equal to the electromotive force:
Vab   (source on open circuit)

The term electromotive force, although widely used, is somewhat unfortunate, in that the
concept to which it refers is not a force but a work per unit charge. The term electromotive is
sometimes used, but the concept is usually referred to simply as emf, pronounced “ee-em-
eff,”

The SI unit of En is the same as that of Ee , namely, one volt per meter, so the unit of emf is
the same as that of potential or potential difference, namely, 1 V. However, an electromotive
force is not the electrostatic field and the former is the work of a non-electrostatic field.

As we shall see later, the electrostatic field within a source, and hence the potential
difference between its terminals, depends on the current in the source. The non-electrostatic
field, and hence the emf of the source, is in many cases a constant independent of the
current, and hence the emf represents a definite property of a source. Unless stated
otherwise, we shall assume, in what follows, that the emf of a source is constant.

Now suppose that the terminals of a source are connected by a wire, as shown schematically
in above figure forming a complete circuit. The driving force on the free charges in the wire is
due solely to the electrostatic field Ee set up by the charged terminals a and b of the source.
This field sets up a current in the wire from a toward b. The charges on the terminals
decrease slightly and the electrostatic fields, both within the wire and within the source,
decrease also. As a result, the electrostatic field within the source becomes smaller than the
(constant) non-electrostatic field. Hence positive charges within the source and driven toward
the positive terminal, and there is a current within the source from b towards a. The circuit
settles down to a steady state in which the current is the same at all cross sections.
CURRENT ELECTRICITY 38

If current could travel through the source without impediment (that is, if the source
had no internal resistance), charge entering the external circuit through terminal a would be
replaced immediately by charge flow through the source. In this case the internal
electrostatic field in the source would not change under complete-circuit conditions, and the
terminal potential difference Vab would still be equal to  . Since Vab is also related to the
current and resistance in the external circuit by
  IR ,
Where R is the resistance of the external circuit. This relation determine the current in the
circuit, once  and R are specified.

Internal Resistance (r):


We say “if” in the above paragraph because every real source has some internal
resistance, which we may denote as r. Under closed-circuit conditions the total electric field
Ee  En inside the source cannot then be precisely zero because some net field is required in
order to push the charge through the internal resistance. Thus Ee must have somewhat
smaller magnitude than En , and correspondingly Vab is less than  ; the difference is equal
to the work per unit charge done by the total field, which is simply Ir . Thus the terminal
potential difference under closed-circuit conditions is given by
Vab    Ir
Where r is the internal resistance of the source. The equation determining the current in the
complete circuit is then

  Ir  IR or I
Rr
That is, the current equals the source emf divided by the total circuit resistance, external
plus internal.
If the terminals of a source are connected by a conductor of zero (or negligible) resistance, the
source is said
to be short circuited. (This would be an extremely dangerous procedure to carry out with the
storage battery of your car, or with the terminals of the power line!) Then R=0, and from the
circuit equation the short circuited current Is is

Is 
r
The terminal voltage is then zero.

Vab      r  0
r
The electrostatic field within the source is zero, and the driving force on the charges within it
is due to the non-electrostatic field only.

A source is completely described by its emf  and its internal resistance r. These
properties may be found (at least in principle) from measurements of the open circuit
terminal voltage, which equals  , and the short-circuit current, which enables r to be

calculated from Eq. Is 
r
39 CURRENT ELECTRICITY
We must consider one more special case. If a source is connected to an external circuit
containing other sources, it is possible that the electrostatic field within the given source will
be greater than the non-electrostatic field, as in figure below. When this is the case, the
current within the source is from terminal a toward terminal b. This is the case when the
storage battery of an automobile is being “charged” by the alternator. Equation Vab    Ir ,
then becomes.
Vab    Ir
The terminal voltage is then greater than the emf  .

Note:
1) A variable resistor is called a rheostat or, particularly in electronics, a potentiometer. A
common type consists of a resistor with a sliding contact that can be moved along its length
is represented by the symbol.

Connections are made to either end of the resistor and to the sliding contact. The symbol.

is also used for a variable resistor.


2) A source with internal resistance is represented by the symbol given below.
CURRENT ELECTRICITY 40

POTENTIAL DIFFERENCE (V) ACROSS THE TERMINALS OF A CELL

Situation potential difference (v)


across the terminal of cell
VAB    ir or VAB  

VAB    ir or VAB  

VAB   as i  0


i or   ir
r
   ir  0 or V  0

LEARN THE SKILLS


31. a) A car has a fresh storage battery of emf 12 V and internal resistance 5.0  10 2  . If the
starter motor draws a current of 90 A, what is the internal voltage of the battery when the
starter is on?
b) After long use, the internal resistance of the storage battery increases to 500  . What
maximum current can be drawn from the battery? Assume the emf of the battery to remain
unchanged.
c) If the discharged battery is charged by an external emf source, is the terminal voltage of
the battery during charging greater or less than its emf 12 V?
Sol. We have emf of the storage battery as   12 V , internal resistance of the battery as
r  0.5 102  , and current flowing through the circuit as I  90 A .


a) If V is the terminal voltage, V    Ir  12  90 5.0 102  7.5 V
b) We know that I   /  R  r  , where R is the external resistance. For I to be maximum
 i.e.,Imax  , R=0 (i.e., the battery is to be shorted). Thus,
 12V
Imax    24 mA (as after long use, r  500  )
r 500 
Clearly, the battery can now no longer to be used for starting the car.
c) During charging, the current inside the battery flows in a direction opposite to that when it
is discharging.
Clearly, replacing I by-I, we get V     I  r    Ir .
Hence, V should be greater than    12 V  during charging.
Energy Transfer In An Electric Circuit
41 CURRENT ELECTRICITY

Figure shows a simple circuit in which a resistor CD having a resistance R is connected to a


battery of emf  through two connecting wires CA and DB. The connecting wires are assumed
to have negligible resistance. This ensures that potential difference across AC and across BD
are zero even when there is a current through them. The potential difference across the
resistor is the same as that across the battery. If the current in the circuit is i, this potential
difference is
V  VA  V B  VC  VD  iR

Thermal Energy Produced in the Resistor


In time t, a charge q=it goes through the circuit. As this charge moves from C to D, the
electric potential energy decreases by
U  qV   it  iR   i2 Rt
This loss in electric potential energy appears as increased thermal energy of the
resistors. Thus, a current i for a time t through a resistance R increases the thermal energy
by i2 Rt . The power developed is
U
P  i2 R
t
Using Ohm’s law, this can also be written as
V2
P  Vi
R
Resistors in series and in parallel
i) Resistors In Series

Figure represents a circuit consisting of a source of emf and two resistors connected in
series. We are interested in finding the resistance R of the network lying between A and B.
i.e., what single equivalent resistor R would have the same resistance as the two resistors
linked together.
Because there is only one path for electric current to follow, i must have the same
value everywhere in the circuit. The potential difference between A and B is V. This potential
difference must some how be divided into two parts V1 and V2 as shown.
Subject to the condition
V  V1  V2  iR1  iR2 or V  i  R1  R2  …………….(i)
Let R be the equivalent resistance between A and B, then
V  iR ………………..(ii)
CURRENT ELECTRICITY 42

From Eqs. (i) and (ii), we get


R  R1  R2 (for resistors in series)
This result can be readily extended to a network consisting of n resistors in series.
R  R1  R2  .....  Rn

Distribution Of Potential In Series Connections


When more than one resistance are connected in series, the current through them is same
and the potential is distributed in the ratio of their resistance, as
V  iR or VR (for same value of i)
For e.g. in the figure.
V1 : V2 : V3  R : 2R : 3R  1: 2 : 3

 1  V  2  V  3  V
V1   V   V2   V  and V3   V 
1  2  3  6 1  2  3  3 1  2  3  2

ii) Resistors In Parallel

In figure, the two resistors are connected in parallel. The voltage drop across each resistor is
equal to the source voltage V. The current i, however, divides into two branches, which carry
currents i1 and i2 .
i  i1  i2 …………..(iii)
If R be the equivalent resistance, then
V V V
i  , i1  and i2 
R R1 R2
Substituting in Eq. (iii), we get
1 1 1
  (for resistors in parallel)
R R1 R2
This result can also be extended to a network consisting of n resistors in parallel. The result
is,
1 1 1 1
   .............. 
R R1 R2 Rn
Distribution Of Current In Parallel Connections
43 CURRENT ELECTRICITY
When more than one resistances are connected in parallel, the potential difference
across them is equal and the current is distributed among them in inverse ratio of their
resistance, as

V 1
i or i (for same value of V)
R R
e.g. in the figure.
1 1 1
i1 : i2 : i3  : :  6:3:2
R 2R 3R

 6  6  3  3  2  2
 i1   i  i , i2   i  i and i3   i  i
 6  3  2  11  6  3  2  11  6  3  2  11

LEARN THE SKILLS


32. The V-I graphs for two resistors and their series combination are shown in figure. Which one
of these graphs represents the series combination of the two resistors? Give reason for your
answer.

Sol. From,
V V V
R1  , R2  ,R3  , Since, I1  I2  I3 , so R1  R2  R3
I1 I2 I3
The resistance of the series combination, being the sum of the two resistances, is greater
than each of the resistances. Since R1  R2, R1  R3 , graph 1 represents the series
combination of two resistors.

33. Two students perform experiments on series and parallel combinations of two given resistors
R1 and R2 and plot the following V-I graphs. Which of the graphs is/are correctly labeled in
terms of the words ‘series’ and ‘parallel’? Justify your answer.
CURRENT ELECTRICITY 44

Sol. We know that the resistance  Rs  in series combination of two resistance is more than the
resistance Rp  in parallel combination of the same two resistors. As I=V/R, for a given

potential  V  , Is  Ip . In both the graphs, for a given potential difference (V) across each
combination, Is  Ip . Hence, both the graphs are properly labeled.

34. In the circuit given, find the currents I, I1 and I2 in the circuit.

Sol. The resistances R1 and R2 are connected in parallel.

63 30
R'   2  and I   6A
63 32
 R2   6   R1   6 
 I1  I    6   4A and I2  I    6   2A
 R1  R 2  3  6  R1  R 2  3  6

35. In the circuit shown in figure, find the heat developed across each resistance in 2 s?

Sol. The 6  and 3  resistances are in parallel. So, their combined resistance is
45 CURRENT ELECTRICITY

1 1 1 1
   or R  2 
R 6 3 2
The equivalent simple circuit can be drawn as shown in figure.

Current in the circuit,


net emf 20
i   2 A and V  iR   2 2  4 V
total resistance 3  2  5
i.e., potential difference across 6  and 3  resistances are 4 V. Now,
2 2
H3  i2Rt   2  3  2  24 J , H5  i2Rt   2 5  2  40 J
2 2
V2  4 16 V2  4   2  32 J
H6  t  2  J and H3  t
R 6 3 R 3 3

Kirchhoff’s Laws
Many electric circuits cannot be reduced to simple series-parallel combinations. For
example, two circuits that cannot be so broken down are shown in figure.

However, it is always possible to analyse such circuits by applying two rules, devised
by Kirchhoff’s in 1845 and 1846 when he was still a student.
First here are two terms that will use often.

Junction
A junction in a circuit is a point, where three or more conductors meet. Junctions are
also called nodes or branch points.

For example, in figure (a) points D and C are junctions. Similarly in figure. (b) points B and F
are junctions.

Loop
A loop is any close conducting path. For example, in figure (a) ABCDA, DCEFD and
ABEFA are loops. Similarly, in figure, (b), CBFEC, BDGFB are loops.
Kirchhoff’s rules consist of the following two statements.
Kirchhoff’s Junction Rule
The algebraic sum of the currents into any junction is zero.
CURRENT ELECTRICITY 46

i.e,  i0
Junction
This law can also be written as, the sum of all the currents directed towards a point in
a circuit is equal to the sum of all the currents directed away from that point.
Thus, in figure i1  i2  i3  i4

The junction rule is based on conservation of electric charge. No charge can accumulate at a
junction, so the total charge entering the junction per unit time must equal to charge leaving
per unit time. Charge per unit time is current so, if we consider the currents entering to be
positive and those leaving to be negative, the algebraic sum of currents into a junction must
be zero.

LEARN THE SKILLS


36. Calculate the values of currents I1,I2,I3 and I4 in the section of network shown in figure.

Sol. The distribution of currents in different resistances can be calculated using Kirchhoff’s
junction rule. We will apply Kirchhoff’s junction rule at junction A, B, C and D as shown in
figure.

By applying junction rule at junction A, we have I2  8  15  I2  7 A ,


By applying junction rule at junction B, we have I2  3  I3  I3  10 A ,
47 CURRENT ELECTRICITY

By applying junction rule at junction D, we have, I4  5  8  I4  3 A .


By applying junction rule at junction C, we have I1  I3  I4  I1  13 A ,

37. Figure shows three resistances are connected with switch S initially is open. When the switch
S is closed find the current passed through it.

Sol. Let V be the potential of the junction as shown in figure.

20  V 5  V V  0
Applying junction law, we have i1  i2  i3    or
2 4 2
V
40  2 V  5  V  2V or 5V  45 or V  9V ,  i3   4.5 A
2

38. Find the current through 12  resistor in figure.

Sol. Let V be the potential at P; then applying Kirchhoff’s junction law at junction P, we get

15  V V  2 V  3 V  4 68 15   68 /15  157
I  I1  I2  I3     or V  V and I   A
12 6 4 8 15 12 180
Kirchhoff’s Loop Rule
The algebraic sum of the potential difference in any loop including those associated
emf’s and those of resistive elements, must equal zero.
CURRENT ELECTRICITY 48

i.e.,  V  0
closed loop

Kirchhoff’s second rule is based on the fact that the electrostatic field is conservative
in nature. This result states that there is no net change in electric potential around a closed
path. Kirchhoff’s second rule applies only for circuits in which an electric potential is defined
at each point. This criterion may not be satisfied, if changing electromagnetic fields are
present.

In applying the loop rule, we need sign conventions. First assume a direction for the
current in each branch of the circuit. Then, starting at any point in the circuit, we imagine,
travelling around a loop, adding emf’s and iR terms as we come to them.

When we travel through a source in the direction from negative to positive, the emf’s is
considered to be positive, when we travel from positive to negative, the emf is considered to
be negative.

When we travel through a resistor in the same direction as the assumed current, the
iR term is negative, because the current goes in the direction of decreasing potential. When
we travel through a resistor in the direction opposite to the assumed current, the iR terms is
positive, because this represents a rise of potential.

Note:
1) It is advised to write H (for higher potential) and L (for lower potential) across all the
batteries and resistances of the loop under consideration while using the loop law. Then write
– while moving from H to L and + for L to H. Across a battery write H on positive terminal and
L on the negative terminal. Across a resistance keep in mind the fact that current always
flows from higher potential (H) to lower potential (L). For example, in the loop shown in
figure we have marked H and L across all batteries and resistances. Now, let us apply the
second law in the loop ADCBA. The equation will be
iR2  E2  iR1  E1  0

2)
49 CURRENT ELECTRICITY

In figure (a) there are eight wires and hence, will have eight currents of eight unknowns. The
eight wires are AB, BC, CE, EA, AD, BD, CD and ED. Number of loops are four. Therefore,
from the second law we can make only four equations. Total number of junctions are five (A,
B, C, D and E). But by using the first law, we can make only four equations (one less). So,
the total number of equations are eight.

In figure (b) number of wires are six (AB, BC, CDA, BE, AE and CE). Number of loops are
three so, three equations will be obtained from the second law. Number of junctions are four
(A, B, C and E) so, we can make only three (one less) equations from the first law. But total
number of equations are again six.

3) Short Circuiting: Two points in an electric circuit directly connected by a conducting wire are
called short circuited. Under such condition both points are at same potential For example,
resistance R1 in the circuit below is short circuited, i.e., potential difference across it is zero.
Hence, no current will flow through R1 and the current through R 2 is therefore, E /R2 .

4) Open–Circuit: Two points are said to be open-circuited when there is no direct connection
between them; a break in the continuity of the circuit exists. Due to this break, the
resistance between the two points is infinite and there is no of current between the two
points.

5) Equipotential Points: In a current-carrying electrical network, two points are said to be


equipotential if they are at the same potential. Between the points 1 and 2, V1  V2 , if
V  iR  0 .

Then we have two cases: if R=0, V  0  i  0  and i=0 (R is finite), V  0 . The first
case tells that when we connect any two points by an ideal conductor, the potential difference
between them becomes zero. It is called short-circuiting. The second case tells that if we
connect any two points by a non zero resistor and find no current along the resistor, we call
these points equivalent. After finding the equipotential points, join them to a single point to
simplify the given circuit.
CURRENT ELECTRICITY 50

6) For a current flow through a resistance there must be a potential difference across it but
between any two points of a circuit the potential difference may be zero.
For example, in the circuit, net emf = 3V

and net resistance  6 


 current in the circuit,
3 1
i  A
6 2
1
VA - VB : VA  1  2 
 VB or VA  VB  0
2
or by symmetry, we can say that
VA  VB  VC
So, the potential difference across any two vertices of the triangle is zero, while the current in
the circuit is non-zero.

7) Earthing: If some point of a circuit is earthed then its potential is taken to be zero. For
example, in the adjoining figure,

VA  VB  0 , VF  VC  VD  3V and VE  9V


 VB  VC  9 V
VB  VE 9
or current through 2 resistance is or A
2 2
Similarly, VA  VF  3 V
VA  VF 3
and the current through 4  resistance is or A
4 4
51 CURRENT ELECTRICITY

LEARN THE SKILLS


39. Find currents in different branches of the electric circuit shown in figure.

Sol. HOW TO PROCEED


In this problem there are three wires EFAB, BE and BCDE. Therefore, we have three
unknowns i1 ,i2, and i3 . SO, we require three equations. One equation will be obtained by
applying Kirchhoff’s junction law (either at B or at E) and the remaining two equations, we
get from the second law (loop law). We can make three loops ABEFA, ACDFA and BCDEB.
But we have to choose any two of the them. Further, we can choose any arbitrary directions
of i1 ,i2, and i3 .

Applying Kirchhoff’s first law (junction law) at junction B,


i1  i2  i3 ……….……….(i)
Applying Kirchhoff’s second law in loop 2 (ABEFA),
4i1  4  2i1  2  0 …………………(ii)
Applying Kirchhoff’s second law in loop 2 (BCDEB),
2i3  6  4i3  4  0 …………………(iii)
8 5
Solving Eqs. (i), (ii) and (iii), we get i1  1A , i2 
A and i3   A
3 3
Here, negative sign of i3 implies that current i3 is in opposite direction of what we have
assumed.

40. In the above problem the potential difference between points F and C?
HOW TO PROCEED
To find the potential difference between any two points of a circuit you have to reach from
one point to the other via any path of the circuit. It is advisable to choose a path in which we
come across the least number of resistors preferably a path which has no resistance.
Sol. Let us reach from F to C via A and B,
VF  2  4i1  2i3  V C  VF  VC  4i1  2i3  2
5 4
Substituting, i1  1A and i3   A , we get VF  VC   V
3 3
Here, negative sign implies that VF  VC
CURRENT ELECTRICITY 52

41. In the circuit show in figure, find the current through the branch BD.

Sol. Assume the currents in the circuit as shown in figure. Applying Kirchhoff’s loop rule along
the loop ABDA and moving in the clockwise direction, we get

6I1  3I2  15  0 or 2I1  I 2  5 ………..(i)


Applying Kirchhoff’s loop rule along the loop BCDB, we get
3  I1  I2   30  3I2  0 or I1  2I2  10 ……………………(ii)
Solving Eqs. (i) and (ii) for I2 , we get I2  5 A

42. Find the current in each part of the circuit.

Sol. Let us take the current distribution as shown in the figure.

Apply loop law in figure,


For loop (1), we have 3I  6I1  4.5  0 or I  2I1  1.5 ………(i)

For loop (2), we have 10  I  I1   3  6I1  0 or 10 I  16I1  3 …….(ii)

Solving Eqs. (i) and (ii), we get


1 1 1 1
I A, I1  A, and I  I1    0
2 2 2 2
53 CURRENT ELECTRICITY
43. In the circuit shown in figure, E, F, G, and H are cells of emf 2V, 1V, 3V, and 1 V,
respectively. The resistances 2 ,1,3 , and 1 are their respective internal resistances.

i) Find the potential difference between B and D.


ii) Calculate the potential difference across the terminals of each of the cells G and H.
Sol. The circuit with the currents shown is redrawn in figure.

Applying the loop law to BADB, we have


2i1  2  1  1i1  2  i1  i2   0 or 5i1  2i2  1 ………………(i)

Applying the same law to loop DCBD, we have


3  3i2  1i2  1  2  i1  i2   0 or 2i1  6i2  2 …………………(ii)

From Eqs. (i) and (ii),we get


1 4 5
i1  A, i2   A ,  i1  i2  A
13 13 13
 5  10
i) VD  VB   2    A  V
 13  13
ii) Potential differences across the cell G is
VD  VC  3  3i2  3  3  4 /13   27 /13 V

Potential difference across the cell H is


VB  VC  1  i2  1  4/13  17/13 V  V  E  ir 
CURRENT ELECTRICITY 54

Grouping Of Cells
Cells are usually grouped in following three ways.

a) Series Grouping
Suppose n cells each of emf E and internal resistance r, are connected in series as shown in
figure. Then,

Net emf =nE


Total resistance =nr+R
 Current in the circuit,
net emf nE
i or i
total resistance nr  R

Note:
If polarity of m cells is reversed, then equivalent emf   n  2m  E
While total resistance is still nr+R
 n  2m  E
 i
nr  R

b) Parallel Grouping
Here three cases are possible.

Case (i):
When E and r of each cell has same value and positive terminals of all cells are connected at
one junction while negative at the other.
r
In this situation, the net emf is E. The net internal resistance is as n resistances each of
n
r 
r are in parallel. Net external resistance is R. Therefore, total resistance is   R  and so,
n 
the current in the circuit will be

Net emf E
i or i 
Total resistance r
R
n
55 CURRENT ELECTRICITY

Note:
A comparison of series and parallel grouping reveals that to get maximum current, cells must
be connected in series, if effective internal resistance is lesser than external and in parallel, if
effective internal resistance is greater than external.

Case (ii):
If E and r of each cell are different but still the positive terminals of all cells are
connected at one junction while negative at the other.

Applying Kirchhoff’s second law in loop ABCDEFA,


R E
E1  iR  i1r1  0 or i1  i  1 …………….(i)
r1 r1
Similarly, we can write
R E
i2  i  2 ……………….(ii)
r2 r2
…. …. …. ….
Adding all above equations, we have
1 E
 i1  i2  .........  in   iR     
r r
But i1  i2  .........  in  i
1 E
 i  iR   r    r 
 i
  E / r   Eeq
1  R  1/ r  R eq

Where, Eeq 
  E /r  and R  R  1
 1/r   1/r 
eq

Case (iii):
This is the most general case of the parallel grouping in which E and r of different cells are
different and the positive terminals of few cells are connected to the negative terminals of the
others as shown in figure.

Kirchhoff’s second law in different loops gives the following equations.


CURRENT ELECTRICITY 56

E1 iR
E1  iR  i1r1  0 or i1   ………………..(i)
r1 r1
E iR
E2  iR  i2r2  0 or i2   2  …………………(ii)
r2 r2
E3 iR
Similarly, i3   ………………….(iii)
r3 r3
Adding Eqs, (i), (ii) and (iii), we get
1 1 1
i1  i2  i3   E1 / r1    E2 / r2    E3 / r3   iR    
 r1 r2 r3 
  1 1 1 
or i 1  R        E1 /r1    E2 /r2    E3 /r3 
  r1 r2 r3  
 E1 /r1    E2 /r2    E3 /r3 
 i
1  R 1/r1  1/r2  1/r3 

c) Mixed grouping
The situation is shown in figure.

There are n identical cells in a row and number of rows are m. Emf of each cell is E and
internal resistance is r. Treating each row as a single cell of emf nE and internal resistance
nr, we have,
Net emf=nE
nr
Total internal resistance 
m
Total external resistance =R
 Current through the external resistance R is
nE
i
nr
R
m
This expression after some rearrangements can also be written as,
mnE
i
 
2
mR  nr  2 mnrR
From this expression we see that i is maximum when,
nr
mR  nr or R 
m
or total external resistance = total internal resistance.
Thus, we can say that the current and hence power transferred to the load is maximum
when load resistance is equal to internal resistance. This is known as maximum power
transfer theorem.
57 CURRENT ELECTRICITY

LEARN THE SKILLS


44. n identical cells, each of emf E and internal resistance r, are joined in series to form a closed
circuit. Find the potential difference across any one cell.
Sol. Current in the circuit is
nE E
i 
nr r
The equivalent circuit of one cell is shown figure. Potential difference across the cell is
E
VA  VB  E  ir  E  .r  0
r

45. n identical cells each of emf 6 V, connected in series with an external resistor of 5  , carry a
current of 10 A. If two cells are connected wrongly in series with the same external resistor,
the current flowing through the cells will be 6.45 A. Find the value of n and the internal
resistance of each cell.
Sol. Let r be the internal resistance of each cell. For series connection, Eeff  nE  6n .
nE 6n
reff  nr or i    10 ……………….(i)
nr  R nr  5
For wrong connection of two cells, the effective emf decreases by 2E because the emfs of two
identical cells counteract with each other. Then,
'  nE  E   n  2 E   n  2 6
Eeff
 n  2 E  n  2 6
and '  nr
reff or i ' 
nr  R

nr  5
 6.45 A ………………..(ii)

By solving Eqs. (i) and (ii), we have n = 10 and r  0.1  .

46. Find the emf and internal resistance of a single battery which is equivalent to a combination
of three batteries as shown in figure.

Sol. The given combination consists of two batteries in parallel and resultant of these two in
series with the third one. For parallel combination we can apply,
E1 E2 10 4
 
r1 r2 2 2 3 V
Eeq  
1 1 1 1
 
r1 r2 2 2
1 1 1 1 1
Further,     1  req  1
req r1 r2 2 2
Now this is series with the third one, i.e.,

The equivalent emf of these two is (6 – 3) or 3V and the internal resistance will be,
1  1 or 2
CURRENT ELECTRICITY 58

47. In a mixed grouping of identical cells, five rows are connected in parallel and each row
contains 10 cell. This combination sends a current i through an external resistance of 20  .
If the emf and internal resistance of each cell is 1.5 V and 1 , respectively, then find the
value of i.
Sol. The number of cells in a row is n=10, and the number of such rows is m=5.
nE 10  1.5 15
i    0.68 A.
 nr   10  1  22
R    20  
 m  5 

48. 100 cells each of emf 5 V and internal resistance 1 are to be arranged to produce maximum
current in a 2.5 resistance. Each row contains equal number of cells. Find the number of
rows.
Sol. Total number of cells is
mn=100 …..………..(i)
Current will be maximum when
nr n 1
R or 25  , n  25 m …………….(ii)
m m
From Eqs. (i) and (ii), we get n =50 and m=2.

49. Three cells of emf 3 V, 4 V, and 6 V are connected in parallel. If their internal resistances are
1 , 2 and 1 , find the Eeff ,reff and the current in the external load R  1.6  .

Sol. The cells are connected in parallel. The equivalent cell emf is
3  4 6
  
 Ei / ri 1  2  1
Eeff    2.8 V
1/ ri 1 1 1
 
1 2 1
The internal resistance of the cell is
1 1 1 1 1 5 2
     or reff  
reff ri 1 2 1 2 5

The current in the circuit is


E 2.8
I   1.4 A
Rr 8  2
5 5
59 CURRENT ELECTRICITY
50. In the circuit shown, find

i) VA  VB ii) I1 iii) I2 iv) I


Sol. Method 1:
The given circuit may be replaced by an equivalent battery of emf E and resistance r as
shown in figure.
4  6 
  
4  12  3 1
E  V and r 3
1 1 2 1 1
 
4 12 4 12

Thus, current through the external resistor is (from A to B)


E 3/2 1
I   A
rR 36 6
For potential difference VA  VB
1 1
VA  6   VB or VA  VB  6   1V
6 6
For upper branch:
7
VA  I212  6  VB or  VA  VB   6  I1  12 or 1  6  I1  12 or I1  A
12
For lower branch:
3
VA  I2  4  4  VB or  VA  VB   4  I2  4 or I2  A
4

Method 2:
Instead of making equivalent battery of two branches, we can make equivalent battery
of all three branches. In the middle branch, we can assume a battery of zero emf connected
with resistance 6  . The given circuit may be replaced by an equivalent battery of emf E and
resistance r as shown in figure.
CURRENT ELECTRICITY 60

4  6  0
   
4  12  6 1
E  1V and r  2
1 1 1 1 1 1
   
4 12 6 4 12 6
This battery is an open circuit battery, hence potential difference across each branch will be
1 V. Now we will get the same result as we have in pervious method.

INTEXT QUESTIONS
46. Suppose you have three resistors of 20,50 and 100. What minimum and maximum
resistances can you obtain from these resistors?
Ans. 12.5 , 170.

47. Three bulbs, each having a resistance of 180, are connected in parallel to an ideal battery of
emf 60 V. Find the current delivered by the battery when (a) all the bulbs are switched on, (b)
two of the bulbs are switched on and (c) only one bulb is switched on.
Ans. (a) 1 A (b) 2/3 A

48. Find the minimum number of cells required to produce a current of 1.5 A through a
resistance of 30  . Given that the emf of each cell is 1.5 V and the internal resistance is 1  .
Ans. nm = 120

49. Consider the circuit shown in figure. Find the current through the 10 resistor when the
switch S is (a) opened (b) closed.

Ans. (a) 0.1 A (b) 0.3 A

50. (a) A car has a fresh storage battery of emf 12 V and internal resistance 5.0 102 . If the
starter draws a current of 90 A, what is the terminal voltage of the battery when the starter is
on?
(b) After long use, the internal resistance of the storage battery increases to 500. What
maximum current can be drawn from the battery? Assume the emf of the battery to remain
unchanged.
(c) If the discharged battery is charged by an external emf source, is the terminal voltage of
the battery during charging greater or less than its emf 12 V?
Ans. (a) 7.5 V (b) 24 mA (c) greater than 12 V.
61 CURRENT ELECTRICITY
51. For the resistor network shown in figure, the potential drop between a and b is 12 V.

(a) Current through resistance of 6  is____________.


(b) Current through resistance of 2 is____________.
(c) Current through resistance of 8  is____________.
10 4 3
Ans. (a) A (b) A (c) A
7 7 4

52. For the circuit shown in figure, find the voltage across 10 resistor and the current passing
through it.

25 5
Ans. V  2.78 V, A  0.278A
9 18

53. For the circuit shown in figure, determine the unknown voltage drop V1.

Ans. 19 V

54. A resistor develops 400 J of thermal energy in 10 s when a current of 2 A is passed through
it. (a) Find its resistance. (b) If the current is increased to 4 A, what will be the energy
developed in 20 s.
Ans. (i) 10  (ii) 3200 J

55. Find the current in 10 resistance, V1 and source voltage VS in the circuit shown in figure
 VS  VA  VB 

Ans. 5 A, 74 V, 49 V (+ve terminal is connected at point B)


CURRENT ELECTRICITY 62

56. Compute the equivalent resistance of the network shown in figure and find the current i
drawn from the battery.

Ans. 3A

57. Determine the voltage drop across the resistor R1 in the circuit given below with E = 65 V,
R1  50 , R 2  100 , R 3  100  and R 4  300 . .

Ans. 25V

58. A 20 V battery of internal resistance 1  is connected to three coils of 12  , 6  and 4  in


parallel, a resistor of 5  and a reversed battery (emf 8V and internal resistance 2  ) as
shown in figure. Calculate

(a) the current in the circuit,


(b) current in resistor of 12  coil, and
(c) potential difference across each battery.
Ans. (a) 1.2 A (b) 0.2 A (c) 18.8 V; 10.4 V

59. Potential difference across the terminals of a cell was measured (in volts) against different
currents (in ampere) flowing through the cell. A graph was drawn, which was a straight line
ABC. Using the data given in the graph (see figure), determine.

(a) the emf, and (b) the internal resistance of the cell.
Ans. (a) 1.4 V (b) 5 
63 CURRENT ELECTRICITY
60. When 10 identical cells in series are connected to the ends of a resistance of 59  , the
current is found to be 0.25 A. But when the same cells, being connected in parallel, are
joined to the ends of a resistance of 0.05  , the current is 25 A. Calculate the internal
resistance and the emf of each cell.
Ans. E= 1.5 V, r  0.1

61. The current in a simple series circuit is 5A. When an additional resistance of 2  is
introduced, the current is reduced to 4A. Calculate the resistance of the original circuit.
Assume that the applied potential difference is the same in both the cases.
Ans. 8

62. A parallel combination of an 8  resistor and an unknown resistor R is connected in series


with a 16 resistor and a battery. This circuit is then disassembled, and the three resistors
are then connected in series with each other and the same battery. In both arrangements,
the current through the 8  resistor is the same. What is the unknown resistance R?
Ans. R  128

63. A battery of emf 2V and internal resistance 0.1  is being charged with a current of 5 A. In
what direction will the current flow inside the battery? What is the potential difference
between the two terminals of the battery?
Ans. ve to  ve terminal, 2.5V

64. In the following circuit, the potential difference between P and Q is

Ans. VPQ  5V

65. Consider the circuit shown in figure. The current through the 6 resistor is 4A, in the
direction shown. What are the currents through the 25  and 20  resistors?

Ans. 9.95A

66. In the circuit shown in figure, the voltage across the 2 resistor is 12 V. What is the emf of
the battery and the current through the 6  resistor?

Ans. 3A
CURRENT ELECTRICITY 64

67. In the circuit (figure) the cells E1 and E2 have emfs of 4 V and 8 V and internal resistances
0.5  and 1.0  , respectively. Calculate the current through 6  resistance.

1
Ans. A
6

68. Six lead-acid type of secondary cells, each of emf 2.0 V and internal resistance 0.015, are
joined in series to provide a supply to a resistance of 8.5. Determine: (i) the current drawn
from the supply and (ii) its terminal voltage.
12 12  8.5
Ans. (i)  1.4 A, (ii)  11.9 V
8.59 8.59

69. In the figure each cell has an emf of 1.5 V and internal resistance of 0.40. Calculate:

(i) current I (ii) current in the 36 resistor


(iii) potential difference across A and B.
1 1 1
Ans. (i)  0.5A (ii)  0.0833 A (iii) 1.5   0.4  1.7V
2 12 2

70. In the circuit shown all five resistors have the same value 200 ohms and each cell has an emf
3 volts. Find the open circuit voltage and the short circuit current for the terminals A and B.

Ans. VB  VA  21/5  4.2V, I  35 /2mA  17.5mA  B to A 

71. Find the currents through the three resistors shown in figure

Ans. zero in the upper 4  resistor and 0.2 A in the rest two.

72. Find the value of i1 / i 2 in figure if (a) R  0.1, (b) R  1 (c) R  10. Note from your answer
that in order to get more current from a combination of two batteries they should be joined in
65 CURRENT ELECTRICITY
parallel if the external resistance is small and in series if the external resistance is large as
compared to the internal resistances.

1.2 10.5
Ans. (a)  0.57 (b) 1 (c)  1.75
2.1 6

EQUIVALENT RESISTANCE OR EFFECTIVE RESISTANCE


1) Method Of Successive Reduction
It is the most common method to determine the equivalent resistance. This method is
applicable only when we are able to identify resistance in series or in parallel. The method is
based on the simplification of the circuit by successive reduction of the series and parallel
combinations. This method is explained with the following example.

LEARN THE SKILLS


51. Find the effective resistance between points A and B.

Sol. The given system can be reduced as shown in figure.

2) Balanced Wheatstone Bridge Circuits


CURRENT ELECTRICITY 66

The scientist Wheatstone designed a circuit to find unknown resistance. Such a circuit
is popularly known as Wheatstone’s bridge. This is an arrangement of four resistances which
can be used to measure one of them in terms of the rest.
The figure shows the circuit designed by him. The bridge is said to be balanced when
deflection in galvanometer is zero, i.e., ig  0 , and hence,

VB  VD
i1 R
Under this condition, VA  VB  VA  VD or i1P  i2R or 
i2 P
i1 S
Similarly, VB  VC  VD  VC or i1Q  i2S or 
i2 Q
R S P R
From Eqs. (i) and (ii),  or 
P Q Q S
So, this is a condition for which a Wheatstone bridge is balanced.

To measure the resistance of an unknown resistor, it is connected as one of the four resistors
in the bridge. One of the other three should be a variable resistor. Let us suppose P is the
unknown resistance and Q is the variable resistance. The value of Q is so adjusted so that
deflection through the galvanometer is zero. In this case the bridge is balanced and
R
P    .Q
S
Knowing R, S and Q, the value of P is calculated. Following two points are important
regarding a Wheatstone’s bridge.
(i) In Wheatstone bridge, cell and galvanometer arms are interchangeable.

In both the cases, condition of balanced bridge is


P R

Q S
(ii) If bridge is not balanced current will flow from D to B if,
PS  RQ
Different Forms Of Wheatstone’s Bridge:
67 CURRENT ELECTRICITY
Following are given few circuits which are basically Wheatstone’s bridge circuits.

LEARN THE SKILLS


52. Find the equivalent resistance of the network shown in figure between the points A and B.

Sol. Suppose an ideal battery of emf E is connected across the points A and B. The circuit is a
Wheatstone bridge with the galvanometer replaced by a 50  resistance. As the bridge is
balanced  R1 /R2  R3 /R 4  , there will be no current through the 50  resistance. We can
just remove the 50  resistance without changing any other current. The circuit is then
equivalent to two resistances 30  and 60  connected in parallel. The equivalent resistance
is
 30     60  
R  20 
 30     60  
3) Current Division Method
53. Find the equivalent resistance between the points a and c of the network shown in figure.
Each resistance is equal to r ?

Sol.

Suppose a potential difference V is applied between a and c so that a current i enters at a


and the same current leaves at c. The current i divides in three parts at a. By symmetry, the
part in ad and in ab will be equal. Let each of these currents be i1 . The current through ao is
i  2i1 . Similarly, currents from dc, bc and oc combine at c to give the total current i. Since
the situation at c is equivalent to that at a, by symmetry, the currents in dc and bc will be i1
and that in oc will be i  2i1 .
Applying Kirchhoff’s junction law at d, we see that the current in do is zero. Similarly,
the current in ob is zero. We can remove do and ob for further analysis. It is then equivalent
CURRENT ELECTRICITY 68

to three resistances, each of value 2r, in parallel. The equivalent resistance is, therefore,
2r/3.

54. Twelve wires, each having resistance r, are joined to form a cube as shown in figure. Find the
equivalent resistance between the ends of a face diagonal such as a and c.

Sol. Suppose a potential difference V is applied between the points a and c so that a current i
enters at a and the same current leaves at c. The currents distribution is shown in figure.

By symmetry, the paths ad and ab are equivalent and hence will carry the same current i1 .

The path ah will carry the remaining current i  2i1 (using Kirchhoff’s junction law).

Similarly at junction c, currents coming from dc and bc will be i1 each and from fc will be

i  2i1 . Kirchhoff’s junction law at b and d shows that currents through be and dg will be zero
and hence may be ignored for further analysis. Omitting these two wires, the circuit is
redrawn in figure (b).

 2r  2r 
The wire hef and hgf are joined in parallel and have equivalent resistance r
 2r    2r 
between h and f. This is joined in series with ah and fc giving equivalent resistance
r  r  r  3r . This 3r is joined in parallel with adc (2r) and abc (2r) between a and c.
The equivalent resistance R between a and c is, therefore, given by
1 1 1 1 3
   , giving R r
R 3r 2r 2r 4

55. Find the equivalent resistance of the circuit of the previous problem between the ends of an
edge such as a and b in figure.
69 CURRENT ELECTRICITY
Sol.

Suppose a current i enters the circuit at the point a and the same current leaves the circuit
at the point b. The current distribution is shown in figure. The paths through ad and ah are
equivalent and carry equal current i1 . The current through ab is then i  2i1 .

The same distribution holds at the junction b. Currents in eb and cb are i1 each. The current
i1 in ah is divided into a part i2 in he and i1  i2 in hg. Similar is the division of current i1 in
ad into dc and dg. The rest of the currents may be written easily using Kirchhoff’s junction
law.
The potential difference V between a and b may be written from the paths ab, aheb
and ahgfcb as
V   i  2i1  r , V   i1  i2  i1  r and V  i1   i1  i2   2  i1  i2    i1  i2   i1  r
which may be written as
V   i  2i1  r , V   2i1  i2  r and V   6i1  4i2  r
V 7
Eliminating i1 and i2 from these equations,  r
i 12
Which is the equivalent resistance.

56. Find the equivalent between the points a and b of the circuit shown in figure.

Sol.

Suppose a current i enters the circuit at the point a, a part i1 goes through the 10 resistor
and the rest i  i1 through the 5  resistor. By symmetry, the current i coming out from the
point b will be composed of a part i1 from the 10 resistor and i  i1 from the 5  resistor.
Applying Kirchhoff’s junction law, we can find the current through the middle 5  resistor.
The current distribution is shown in figure.
We have Va  Vb   Va  Vc    Vc  Vb   10   i1   5   i  i1    5   i   5   i1 …..(i)
CURRENT ELECTRICITY 70

Also, Va  Vb   Va  Vc    Vc  Vd    Vd  Vb   10   i1   5   2i1  i   10   i1


   5  i   30  i1 …………………(ii)
Multiple (i) by 6 and subtracting (ii) from it, we eliminate i1 and get,
Va  Vb
   5   i   30   i  7
or
i
Thus, the equivalent resistance between the points a and b is 7  .

57. Find the equivalent resistance of the network shown in figure between the points a and b.

Sol. Suppose a current i enters the network at point a and the same current leaves it at point b.
Suppose, the currents in ac, ad and ae are i1,i2 and i3 respectively. Similar will be the
distribution of current at b. The current i leaving at b is composed of i1 from db, i2 from cb
and i3 from eb. The situation is shown in figure (a).

As the current in ae is equal to that in eb, the current in ce will be equal to the current in ed
from the junction law. If we assume that the branches ced and aeb do not physically touch at
e, nothing will be changed in the current distribution. We can then represent the branch aeb
by a single resistance of 10 connected between a and b. Similarly, the branch ced may be
replaced a single 5  resistor between c and d. The circuit is redrawn in figure. This is same
as the circuit of the figure of the above problem connected in parallel with a resistance of
10 . So the network is equivalent to a parallel combination of 7  and10  resistor. The
equivalent resistance of the whole network is, therefore,
 7    10  
R  4.1 
 7    10  
4) Method Of Same Potential:
This method is based on identifying the points of equal potential and connecting them. By
doing so the electric resistance network reduces to an arrangement of series and parallel
combinations that can be easily solved by the successive reduction method.
71 CURRENT ELECTRICITY

LEARN THE SKILLS


58. Four identical resistances each having value R are arranged as shown in figure. Find the
equivalent resistance between A and B.

Sol. Since C and D are connected with a zero resistor, they are equipotential. Then superimpose
C and D to obtain the simplified circuit as shown. Since no current flows in the branches CE
and ED, cut and then throw them to have
R AB  R  R  2R
If we get a closed loop of resistors without any battery, it carries no current. Then remove the
total loop to get a simpler circuit or if the current in any branchy is zero, remove it.

59. Find the R AB in the given network.

Sol. As A, C and B, D are short-circuited, A and C are at the same potential; B and D are at the
same potential. Brining A and C to one point and B and D to one point, we have redrawn the
circuit. You can see that all resistors are in parallel. Then,

1 1 1 1
   or R  1 
R 3 6 2

METHOD OF SYMMETRY FOR IDENTIFYING POINTS OF SAME POTENTIAL


1) Parallel Axis Of Symmetry:
It is along the direction of current flow. Let us discuss this concept by an example. In the
circuit shown in figure below even though the resistors 1 and 2 do not appear to be
connected in parallel, but they can be treated as parallel, why? For explaining this, we have
CURRENT ELECTRICITY 72

to use the concept of symmetry. Note that the circuit is symmetric about the line MN.
Therefore, all characteristics such as potential and current should also be symmetrical.

From this, it means that current in 1 I1   current in 2  I2  , current in 3  I3   current


in 4  I4  , current in 7  I7   current in 8  I8  , hence the potential difference across 1 is equal
to the potential difference across 2.

Note that the conditions for 1 and 2 to be in parallel are satisfied. Hence, we can
consider them to be in parallel; however, if one of the resistors had a different values, we
cannot use this method.

2) Perpendicular Axis Of Symmetry:


It is perpendicular to the direction of flow of current. Consider the circuit diagram
given in the above figure. The circuit has perpendicular axis of symmetry about XY. The
perpendicular axis of symmetry means that the circuit diagram is symmetric except for the
fact that the input and output are reversed. That is only the flow of current will not be a
mirror image about this particular axis. For example, in the above case, elements 1 and 4 are
symmetric about XY, but the current flow condition is not a mirror image. The current flow
condition is in the same direction.

This implies that current into 6 = current out of 5, current into 1= current out of 4,
etc. Perpendicular axis of symmetry is a very powerful principle. In fact just by looking at the
circuit, we can easily say that (since the circuit has perpendicular axis of symmetry about
XY) no current in flow in elements 7 and 8.

Therefore, we can ignore these two elements completely. In some cases, we may not be
able to use symmetry to simplify the circuit, but we can find out some of the characteristics
of current/potential based on symmetry. We should always look out for these characteristics
and use them as much as possible.
73 CURRENT ELECTRICITY

LEARN THE SKILLS


60. In the given circuits, calculate the resistance between points A and B.

Sol. a) The circuit has both parallel axis and perpendicular axis of symmetry (by symmetry we
mean that the two parts are mirror images of each other).

Method 1:

Let us solve this problem first using the perpendicular axis of symmetry. We observe
the following.
The current is resistance (1) is equal to the current in resistance (3).
The current in resistance (5) is equal to the current in resistance (6).
The current in resistance (2) is equal to the current in resistance (4).

From these observations it is clear that there is no mingling of current from upper and lower
branches into the middle branch. Hence, resistors (7) and (8) are ineffective.

We also observe that all the points on the symmetry line are equipotential. Hence, points b, e,
and d are equipotential points, and so resistance (7) and (8) can be removed. The calculation
of effective resistance is shown in figure.

The current flow is not a mirror image in branches ab and bc because the flow is in
same direction. This is called asymmetric condition. The special thing about this asymmetry
is that current incoming at b is equal to the outgoing current. A similar situation exists at b
and d also. Thus, resistors in branches be and de are ineffective.

Method-2:
CURRENT ELECTRICITY 74

The circuit has parallel axis symmetry about the line ac, so potential and current
must also be symmetrical. Therefore, currents in ab and ad are same. Currents in dc and bc
are also same. Potentials of the points b, e, and d are same. The circuit can be folded about
the parallel axis of symmetry. The calculations of equivalent resistance are shown in figure.

b) The current has parallel axis of symmetry about the line passing through a and b
perpendicular axis of symmetry about the line x-y.

Method-1: From perpendicular axis of symmetry, it is clear that


The current in resistance (7) is equal to the current in resistance (8).
The current in resistance (9) is equal to the current in resistance (10).
The current in resistance (11) is equal to the current is resistance (12).
75 CURRENT ELECTRICITY
From these observations, it is clear that there is no mingling of currents from upper and
lower branch into the middle branch. The upper and lower branches can be separated from
the middle branch. The calculations of equivalent resistance are shown in figure.

Method 2: We can observe the parallel axis symmetry about line AB. The potentials of c
and e and that of d and f should be equal. Hence, the circuit can be folded about the line
passing through A and B. The calculations of equivalent resistance are shown in figure.

As shown in the figure, finally 2R and 4R/3 are in parallel. Hence, the equivalent resistance
will be
4R
2R 
R eq  3  4R
10R 5
3

61. In the network shown in figure. Find the equivalent resistance across the points M and M '

Sol.
CURRENT ELECTRICITY 76

i) The axis MM ' is the parallel axis of symmetry, and the axis NN ' is the perpendicular axis
of symmetry.

ii) Point lying on the perpendicular axis of symmetry may have same potential. In the given
network, points 2, 0, and 4 are at the same potential.

iii) The network can be folded about the parallel axis of symmetry, and the overlapping nodes
have same potential.

Thus, as shown in figure, the following points have same potential: (5 and 6), (2, 0, 4) and (7
and 8). After folding the network about the axis MM ' , the circuit may be simplified by using
the method of successive reduction.

62. Calculate the effective resistance between points A and C, by applying symmetry principle.
77 CURRENT ELECTRICITY

R
Sol. Break the branch AC into two resistors in series, each , and consider the dotted line
2
passing through B, D and E. The network on the two sides of this line is symmetrical. Hence,
one can short-circuit the points B, D and E and calculate R eq  AE .

R
Now, we find that resistances R, R and are in parallel across A and E. The potential
2
R R R R
combination of R and R gives . This in parallel with gives . Hence,
2 2 2 4

 Req AE  R4  Req AC  2 Req AE  2  R4   R2

63. Twelve resistors each of resistance r are connected together so that each lies along the edge
of the cube as shown in figure. Find the equivalent resistance between
a) 1 and 4
b) 1 and 3.

Sol. a) Between 1 and 4: Points 2 and 5 are symmetrically located w.r.t points 1 and 4. So, they are
at the same potential. Similarly, points 3 and 8 are also symmetrically located w.r.t points 1
and 4. So, they are again at same potential. Now we have 12 resistors each of resistance r
connected across 1 and 2, 2 and 3,…, etc. So, redrawing them with the assumption that 2
and 5 are at same potential and 3 and 8 are at same potential. The new figure is shown in
figure.
Now, we had to find the equivalent resistance between 1 and 4. We can now simplify the
circuit as,
CURRENT ELECTRICITY 78

7
Thus, the equivalent resistance between points 1 and 4 is r.
12
b) Between 1 and 3: Points 6 and 8 are symmetrically located w.r.t points 1 and 3. Similarly,
points 2 and 4 are located symmetrically w.r.t points 1 and 3. So, points 6 and 8 are at same
potentials. Similarly, 2 and 4 are at same potentials. Redrawing the simple circuit, we have
figure.

Between 1 and 3 a balanced Wheatstone bridge is formed as shown in figure.


So, the resistance between 2 and 6 and between 4 and 8 can be removed. Hence,

3
Thus, the equivalent resistance between 1 and 3 is r.
4

5) Nodel Method Of Circuit Analysis


This method is explained with following problems.
79 CURRENT ELECTRICITY

LEARN THE SKILLS


64. Find the current in the wire BD in each case.

Sol. a) Let us assign B as reference node and take its potential =0. As nodes B and D are directly
connected with a conducting wire, hence potential of node D will also be zero. Then write the
potential of other nodes; corresponding currents are shown in figure.

10  0   20  0 
I1   5 A and I2  4A
2 5

At node D, I1  I2  I3 or I3  5  4  1 A
Therefore, current in wire BD is 1 A, from D to B.
b) Let us assume the potential of point D to be zero. The potentials at the different points can
be assigned as shown in figure.

From circuit diagram, it is clear that current in 2 resistance will be zero as potential
difference across it will be zero. Hence I1  0 .

20  0
The current through 5  resistance is I2   4 A . Hence, at junction B. I2  I1  I3 . As
5
I1  0 , so I3  I2  4A . Hence, current in wire BD will be 4 A.

c) Let us assume the potential of point B and D be 0 V and xV, respectively. Then the
potential at different points can be assigned as shown in figure.
CURRENT ELECTRICITY 80

 x  20   0  x  10   x  10   0
I1   , I2  and I 3  
5 2 2
 x  20   x  10   x  10 
At node D, I1  I2  I3  0     0 or x=5 V
5 2 2
 x  10  5  10 5
i3     A {from D to B}
 2  2 2
or 5/2 current will flow from B to D.

65. Find the current in each wire,


Given. E1  50 V, E2  40 V
E3  30 V, E4  10 V , R1  2 , R2  2  , R 3  1 

Sol. Let potential at point A be 0 volt, then potential of other points is shown in figure. Analysis of
current in different branches are shown in figure.

0   40 
i1   20 A ,
2
0   30  40  0
i2   15 A , i3   40 A
2 1
and i 4   i1  i2   i3  i4  i3   i1  i2   40   20  15   5 A
66. In the circuit shown in figure. Find the current through the branch BD.
81 CURRENT ELECTRICITY

Sol. Let us assume the potential of node D zero and the potential of node B is x. Then we can
assign the potential at different nodes as shown in figure.

Applying junction law at node B


 x  15   x  30   x  0 
I1  I2  I 3  0     0 or x  15 V
6 3 3
Hence, current through resistance connected across BD is
15  0
i 5 A (from B to D)
3

67. Find the equivalent resistance between A and B

Sol. Let a battery of emf V is connected across terminals A and B and let I be the current through
the source. Then R eq  V /I ,
From symmetry, it is clear that the current in the resistances connected across nodes 1
and 2 will be same as the resistance connected across nodes 5 and 4. Hence, potential
differences across there resistances will be equal.

We can state the same for the resistance connected across the nodes 6 and 5 and nodes 2
and 3. Let us assign the potentials of points A and B to be V and 0, respectively, then we can
assign potentials of different nodes as shown in the figure. At node 2
x  V  x   V  x   x  0  4
  0 or x V …………(i)
5 5 10 7
At point A, using Kirchhoff’s junction rule
CURRENT ELECTRICITY 82

V  x  V   V  x  2V  x
I   …………(ii)
5 10 10
From (i) and (ii),
V
 7   R eq
I

6) The Principle Of Superposition Or Superposition Theorem


Complex network problems can sometimes be solved easily be using the principle of
superposition. This principle essentially states that when a number of emf’s acts in a
network, the solution is the same as the superposition of the solutions for one emf acting at a
time, the others being shorted.

Figure shows a network with two loops. The currents in various branches can be calculated
using Kirchhoff’s Laws. We can get the same solution by considering lonely one battery at a
time and then superposing the two solutions. If a battery has an internal resistance, it must
be left in place when the emf of the battery is removed. Figure shows how the superposition
principle can be applied to the present problem.

The current values in figure (a) and (b) are easily verified. For example when the 21.6
V battery alone is acting, the total resistance in the circuit is,
24  16
8  4  21.6 
24  16
21.6 V
This makes the total current  1 A . This current splits between 16  and 24  in the
21.6 
ratio 3:2.
Similarly, the total resistance when only the 28.8 V battery is acting is,
24  12
16   24 
24  12
28.8V
Therefore, the total current is  1.2 A
24 
The superposition principle shows that there is not current in the 24  resistance. Only a
current of 1.8 A flows through the outer loop. All these conclusions can also be verified by
analyzing the circuit using Kirchhoff’s Laws.
83 CURRENT ELECTRICITY

Note:
Whenever a cell or a battery is present in a branch, there must be some resistance (internal
or external or both) present in that branch in order to apply the superposition principle. In
practical situations it always happens because we can never have an ideal cell or a battery
with zero resistance.

LEARN THE SKILLS


68. In the circuit shown in figure. Find the current through the branch BD. Solve this problem
using superposition principle.

Sol. In superposition method, we can take the effect of one battery at a time. From figure, it can
clear that the effect of two batteries can be taken by adding the effect of individual batteries.

For figure (a)


33 3 15  3  1 
req    and I '   2  , Here I1  I '    2 2  1 A
33 2 3  3  3   
6
2
For figure (b)
63 30  6  6
r 'eq   2  and I ''   6 A , Here I2  I ''    6   4 A
63  3  2 3  6 9
 Current through resistance connected across B and D is i1  i2  5 A (from B to D)
CURRENT ELECTRICITY 84

69. Solve above problem using equivalent battery method.


Sol. Using Equivalent Battery Method (i)
We can treat 6  and 3  resistance connected with 15 V and 30 V batteries,
respectively, as internal resistance of the batteries. Now we can assume these batteries are
connected in parallel, and equivalent battery of these two is connected with resistance
connected across B and D.

15 30

Eeq  6 3  25 V and r  1  2 
eq 1 1
1 1
 
6 3 6 3
Hence, current through resistance connected across B and D is
20
i 5 A
23

Using Equivalent Battery Method (ii)


We can assume a battery of zero emf is connected in series with resistance connected
across BD. This resistance can be treated as internal resistance of zero emf battery. Now we
have three batteries in parallel. The equivalent of these batteries will be a single battery
which is the open circuit.

15 30 0
  1 6
eq  6 3 3  15 V and r 
eq  
1 1 1 1 1 1 5
   
6 3 3 6 3 3
As equivalent battery is an open circuit, the potential difference across each of the battery
branch will be 15 V. Hence, current through the resistance connected across BD is
15
i 5A
3
85 CURRENT ELECTRICITY

7) DELTA-STAR TRANSFORMATIONS
i) Delta To Star Transformation:
Suppose we are given three resistances R12,R23 andR13 connected in delta fashion between
terminals 1, 2 and 3 as in figure (a). These three given resistances can be replaced by the
three resistance R1,R2 and R3 connected in star as shown in figure (b).

These two arrangements will be electrically equivalent because resistance as measured


between any pair of terminals is the same in both the arrangements.
R12R13 R23R12 R13R23
R1  , R2  and R3 
R12  R23  R13 R12  R 23  R13 R12  R23  R13

How To Remember Delta To Star Transformation


Resistance of each arm of the star is given by the product of the resistances of the two
delta sides that meet at its end divided by the sum of the three delta resistances.

LEARN THE SKILLS


70. Find the equivalent resistance of the circuit between the points A and B.

Sol. For finding R AB , we will convert the delta CDE of figure into its equivalent star as shown in
figure.
CURRENT ELECTRICITY 86

4 16 6 24 4 12
R CS  8   ; R ES  8    and R DS  6   .
18 9 18 9 18 9
35
The two parallel resistance between S and B can be reduced to a single resistance of 
9
As seen from figure
 16   35  87
R AB  4       
 9   9  9

ii) Star To Delta Transformation


Suppose we are given three resistances R1, R2 andR3 connected in star fashion between
terminals 1, 2 and 3 as shown in figure (a). These three resistances can now be replaced by
the three resistances R12,R23 andR13 connected in the delta network as shown in figure(b).

These two arrangements will be electrically equivalent because resistance as measured


between any pair of terminals is the same in both the arrangements.
RR R R RR
R12  R1  R2  1 2 , R23  R2  R3  2 3 and R13  R1  R3  1 3
R3 R1 R2

How to remember star to delta


The equivalent delta resistance between any two terminals is given by the sum of star
resistances between those terminals plus the product of these two star resistances divided by
the third star resistance.

LEARN THE SKILLS


71. A network of resistances is formed as follows:
AB  9  , BC  1 , CA  1.5  forming a delta and AD  6  , BD  4  and CD  3 
forming a star. Compute the network resistance measured between
a) A and B, b) B and C, and c) C and A.

Sol.
87 CURRENT ELECTRICITY

The above star may be converted into the equivalent delta and combined in parallel with the
given delta ABC. The three equivalent delta resistance of the given star becomes as shown in
figure.

9  27  27  9 
 6    6  
6  2.25 18 10  20  441 20  10  621
a) R AB    b) R BC    c) R CA   
 6  2.25  11  9
 6 
27  550  9
 6 
27  550
   
 10 20   10 20 
8) Infinite Ladder And Grid
Some networks make a ladder (or a grid) and extend to infinity. To reduce such
networks we use the following steps.
Step 1: Let us assume the total resistance of the infinite network to be X (say).
Step 2: Now just retain one basic repetitive unit and we observe the remaining circuit to be
the same as the original circuit. So, resistance of this left out circuit must be X.
Step 3: Now the equivalent circuit, is the combination of basic unit and original repetitive
circuit of resistance X, such that the net resistance of the entire circuit is X.
The following illustrations are done on the basic of these three steps.

LEARN THE SKILLS


72.

a) Find effective resistance between points A and B of an infinite chain of resistors joined as
shown in figure (a).
b) For what value of R 0 in the circuit shown in figure (b) will the net effective resistance is
independent of the number of cells in the chain?
Sol. a) Suppose the effective resistance between A and B is X.
CURRENT ELECTRICITY 88

Applying the steps discussed

we get,  X  R1 
R2X
 R2  X 
 X2  R1X  R1R2  0  X
1
 R1 
2
 R12  4R1R2  
And as resistance can not be negative, we have
R1  4R 2 
X 1  1  
2  R1 
However, if R1  R2  R we get

X
R
2
 
1  5  1.6 R

b) Suppose there are n sections between points A and B and the network is terminated by
R 0 with equivalent resistance X. Now, if we add one more to the network between the C and
D, the equivalent resistance of the network X will be independent of number of cells if the
resistance between points C and D still remains R 0 (or X). So, the circuit reduces to an
equivalent circuit as shown.

R 2R 0 1 4R 2 
 R1  R  X  R0  1  1  
 R2  R0  0 2 R1 

If R1  R2  R , then we get, X  R 0 
2
R
 
1  5  1.6 R

73. The circuit diagram shown consists of a large number of elements (each element has two
resistors R1 and R2 ). The resistance of the resistors in each subsequent element differs by a
1
factor of k  from the resistances of the resistors in the previous elements. Find the
2
equivalent resistance between A and B shown in figure.
89 CURRENT ELECTRICITY
Sol. When each element of circuit is multiplied by a factor k then equivalent resistance also
becomes k times. Let the equivalent resistance between A and B be X.

So, equivalent circuit becomes

1
X
 R1  R 2   R12  R 22  6R1R 2
For k 
2 2

INTEXT QUESTIONS
73. In the circuit shown in figure potential difference between point A and B is 16 V. Find the
current passing through 2 resistance.

Ans. 3.5A

74. Find the current supplied by the battery in the circuit shown in figure.

Ans. 5A

75. Calculate battery current and equivalent resistance of the network shown in figure.

8
Ans. 15 A, 
5
76. Compute total circuit resistance and battery current as shown in figure.
CURRENT ELECTRICITY 90

8
Ans. ,9A
3

77. Compute the value of battery current i shown in figure. All resistance are in ohm.

Ans. 6A

78. Calculate the potentials of point A, B, C and D shown in figure. What would be the new
potential values if connections of 6- V battery are reversed? All resistances are in ohm.

Ans. VA  12 V,VB  9 V,VC  3 V,VD  6 V


V 'A  12V,V 'B  11V,V 'C  9V,V 'D  6V

79. In the circuit shown in figure, calculate the following:


(i) Potential difference between points a and b when switch S is open.
(ii) Current through S in the circuit when S is closed.

Ans. (i) Vab  12V (ii) 3 amp from b to a


80. In the circuit shown in figure, calculate the current i1, i2, and i3 . Also find out the potential
differences between the points A and B and B and C.
91 CURRENT ELECTRICITY

108 212 10 404 354


Ans. i1  A, i2  A, i3  A, VAB  V, VBC  V
77 77 77 77 77

81. In the circuit shown in figure, calculate (a) current I (b) current I1 and (c) VAB . All
resistances are in ohms.

Ans. a) 4A b) 0.25 A c) 4 V

82. In the circuit shown in figure, V1 and V2 are two Voltmeters having resistances 6000 ohms
and 4000 ohms respectively. E.M.F. of the battery is 250 Volts, having negligible internal
resistance. Two resistances R1 and R2 are 4000 ohms and 6000 ohms respectively. Find the
reading of the voltmeters V1 and V2 when
a) the switch S is open and
b) the switch S is closed.

Ans. a) 150 V, 100 V b) 125 V, 125 V

83. In following circuit potential at point ‘A’ is zero then determine

(a) Potential at each point


(b) Potential difference across each resistance
(c) Identify the batteries which act as a source
(d) Current in each battery
(e) Which resistance consumes maximum power.
(f) Which battery consume or gives maximum power.
Ans. (a) VA  VB  VC  VD  0V, VE  VF  VG  VH  10V, VI  VJ  VK  15V
CURRENT ELECTRICITY 92

(b) V1  15V, V2  5V, V3  15V


(c) each act as a source
(d) 17.5 A (  ), 15A(  ) 2.5 A (  ), 5A (  ) from left to right in given circuit.
(e) 1  resistance
(f) left most battery.

84. Give the magnitude and polarity of the following voltages in the circuit of figure:
(i) V1 (ii) V2 (iii) V3 (iv) V3 2 (v) V12 (vi) V1 3 .

Ans. – 75 V, – 50 V, 125 V, 175 V, – 25 V, 200 V

85. In the circuit, a voltmeter reads 30 V when it is connected across 400  resistance. Calculate
what the same voltmeter will read when it is connected across the 300  resistance.

Ans. 22.5 V

86. Calculate the value of the electric currents I1,I2 and I3 in the given electrical network.

(a) I1  __________ (b) I2  __________ (c) I3  ___________


18 66 48
Ans. (a) (b) (c)
31 31 31
93 CURRENT ELECTRICITY
87. Find the current in each resistor in the circuit as shown in figure.

Ans. I1  3A, I2  2A

88. Determine the currents I1, I2 and I3 for the network shown below (figure).

(a) I1  __________ (b) I2  __________ (c) I3  ___________


Ans. (a) I1  3A, (b) I2  1.5A , (c) I3  4.5A

89. For the circuit in figure, find the potential difference between points a and b.

Ans. 2.4V

90. In the network of three cells, find the potential V of their function.

44
Ans. V V
7

91. Find the current in each wire.


CURRENT ELECTRICITY 94

Ans. EF  1A, BE  10.5 A, DE  11.5A

92. In the circuit shown, all the side resistances are of value 2 and all inner resistances are of


value 4  . Find the current via branch R '. R '  4  

Ans. 0.833 A

93. Find the current I & voltage V in the circuit shown.

Ans. I  2.5A, V  3.5Volts

94. In the circuit shown in figure, find

a) the current in the 3.00  resistor.


b) the unknown emf’s E1 and E2
c) the resistance R.
Ans. a) 8.00 A b) 36.0 V, 54.0 V c) 9.00 
95. Find the current in each branch of the circuit shown in figure.
95 CURRENT ELECTRICITY

Ans.
Resistance 5 8 6 16 4 1
Current 2A 0.25 A 1.5 A 0.25 A 0.5 A 2A
Towards A C C C B E

96. The given wheat stone bridge is showing no deflection in the galvanometer joined between the
points B and D in figure. Calculate the value of R.

Ans. R  25 

97. Calculate the resistance between the terminals A and B of the network shown in figure.

2
Ans. Rp  
3

98. Find the current supplied by the source in figure. The resistors are mounted around in a
cylindrical form.

Ans. 3A
99. Find the current in each resistor of the circuit shown in figure.
CURRENT ELECTRICITY 96

(a) Current through resistance of 6  is __________.


(b) Current through resistance of 3  is __________.
(c) Current through resistance of 2 is __________.
(d) Current through resistance of 5  is __________.
19 1 7 26
Ans. (a) A (b) A (c) A (d) A
6 3 2 5

100. Given that a current of 5.0 A passes along the branch from C to B in figure. What is the
voltage of points A, D, and G?

(a) VA  __________ (b) VD  __________ (c) VG  _______


Ans. (a) 14 (b) 0 (c) 0 V

101. Each resistance in the circuit (figure) is of 2000 .

The combination is put across a supply of 200 V. Find the reading of ammeter.
Ans. 0.4A

102. Consider the circuits shown in figure. Both the circuits are taking same current from the
battery, but the current through R in the second circuit is 1/10 th of the current through R in
the first circuit. If R is 11, then find the value of R1 and R 2 .

Ans. 9.9
103. Calculate the current through the resistance connected across. MN and the current supplied
by each of the batteries in the circuit diagram shown in figure.
97 CURRENT ELECTRICITY

Ans. 4A

104. In the following circuit E1  4 V, R1  2 , E2  6 V,R 2  2, and R 3  4 . Find the currents
i1 and i2 .

Ans. 1.8A; 1.6A

105. Consider the circuit shown in figure. The current i3 is equal to

5
Ans.  A
6

106. In the circuit shown in figure,

(a) find the current in resistor R


(b) find the resistance R
(c) find the unknown emf 
(d) if the circuit is broken at point x, what is the current in resistor R?
Ans. (a) 2A (b) R  5 (c)   42V (d) 3.5A
107. In the circuit shown in figure,
(a) find the current in the 3 resistor
(b) find the unknown emfs 1 and 2
CURRENT ELECTRICITY 98

(c) find the resistance R

Ans. (a) 8A, (b) 36V, 54V (c) 9 

108. In the circuit shown in figure,


(a) find the current in each branch
(b) find the potential difference Vab of point a relative to point b

Ans. (a) I1  0.8A, , I2  0.2A, and I3  0.6A (b) 3.2V

109. Find the current through 25V cell & power supplied by 20V cell in the figure shown.

Ans. 12A,  20W

110. For what value of R in circuit, current through 4 resistance is zero.

Ans. 1

111. In the circuit shown in figure the reading of ammeter is the same with both switches open as
with both closed. Then find the resistance R. (ammeter is ideal)

Ans. 600 
112. If the switches S1,S2 and S3 in the figure are arranged such that current through the battery
is minimum, find the voltage across points A and B.
99 CURRENT ELECTRICITY

Ans. 1V

113. A network of resistance is constructed with R1 & R 2 as shown in the figure. The potential at

the points 1, 2, 3.., N are V1,V2 ,V3 ....,Vn respectively


each having a potential k time smaller than previous one. Find:
R R
(i) 1 and 2 in terms of k
R2 R3
(ii) current that passes through the resistance R 2 nearest to the V0 in terms V0 , k & R 3 .

Ans. (i)
(k  1)2 k
(ii)
(k  1) k  V
2
0
; 3
k (k  1) R

114. An enquiring physics student connects a cell to a circuit and measures the current drawn
from the cell to I1. When he joins a second identical cell is series with the first, the current
becomes I2 . When the cells are connected are in parallel, the current through the circuit is I3 .
Show that relation between the current is 3I3I2  2I1  I2  I3 

115. Find the potential difference VA  VB for the circuit shown in the figure.

22
Ans.  V
9
2
116. Find the current through  resistance in the figure shown.
3

Ans. 1A
117. An electrical circuit is shown in the figure. Calculate the potential difference across the
resistance of 400 ohm, as will be measured by the voltmeter V of resistance 400 ohm, either
by applying Kirchhoff’s rules or otherwise.
CURRENT ELECTRICITY 100

Ans. 20/3V

118. In the circuit shown in fig. E1  3volt,E2  2volt,E3  1volt and R  r1  r2  r3  1ohm.
(i) Find potential difference between the points A and B and the currents through each
branch with A & B un-connected.
(ii) If r2 is short circuited and the point A is connected to point B through a zero resistance
wire, find the currents through E1,E 2 ,E 3 and the resistor R.

Ans. (i) 2 volt, i1  1A, i2  0 A, i3  1A (ii) i1  1A, i2  2 A, i3  1A,iR  2A

119. In the circuit shown in fig. E, F, G and H are cells of emf 2, 1,3 and 1 volts and their internal
resistances are 2, 1, 3 and 1 ohm respectively. Calculate.

(i) The potential difference between B and D and


(ii) The potential difference across the terminals of each of the cells G and H.
2 21 19
Ans. (i) V (ii) V, V
13 13 13

120. In the given circuit the ammeter A1 and A 2 are ideal and the ammeter A 3 has a resistance of
1.9 103 . Find the readings of all three meters.

Ans. I1  82/27, I2  34 /27, I3  0


121. (a) Determine the potential difference between X and Y in the circuit shown in Figure.
101 CURRENT ELECTRICITY

(b) If intermediate cell has internal resistance r  1 then determine the potential difference
between X and Y.
Ans. (a) 3.7V (b) 3.7 V

122. Find the equivalent resistance of the circuit given in figure between the following points:

(i) A and B (ii) C and D (iii) E and F (iv) A and F (v) A and C
Ans. (i) R AB  5 /6 (ii) R CD  1.5  (iii) R EF  1.5  (iv) R AF  5 /6  (v) R AC  4 /3 

123. Find R AB in the circuit, shown in figure.

Ans. 22.5 

124. Find the equivalent resistance between A and B in the following circuits.

42 R 32 25 122 5R 5
Ans. a)  b) c)  d)  e)  f)  g)  h) 4
31 2 21 6 21 4 7
CURRENT ELECTRICITY 102

125. Find the effective resistance of the network (see figure) between the points A and B.
Where R is the resistance of each part.

Ans. 8/7R

126. The figure shows a network of resistor each heaving value 12. Find the equivalent resistance
between points A and B.

Ans. 9

127. If each of the resistance in the shown network is R, then what is the resistance between A
and B.

Ans. R AB  R.

128. Find the equivalent resistance of the following group of resistances between A and B. Each
resistance of the circuit is R

(a) (b)
Ans. (a) 5/7 R, (b) 9R/14
129. In figure shown we wish to determine current in one of the 4 resistors in the circuit.

Ans. zero

130. An infinite ladder network of resistance is constructed with 1 and 2 resistance, as shown
in figure.
103 CURRENT ELECTRICITY

(i) Show that the effective resistance between A and B is 2 .


(ii) What is the current that passes through the 2 resistance nearest to the battery?
Ans. (ii) 1.5 A

131. Consider an infinite ladder network shown in fig. A voltage is applied between points A and
B. If the voltage is halved after each section, find the ratio R1 /R 2 . Suggest a method to
terminate it after a few sections without introducing much error in attenuation.

1
Ans.
2

132. What will be the change in the resistance of a circuit consisting of five identical conductors if
two similar conductors are added as shown by the dashed line in figure?

R2 3
Ans. 
R1 5

133. Find the equivalent resistance of the triangular bipyramid between the points
a) A and C b) D and E
Assume the resistance of each branch to be R.

2 2
Ans. a) R b) R
5 3
134. A square pyramid is formed by joining 8 equal resistances R across the edges. The square
base of the pyramid has the corner at A, B, C, D. The vertex is at M. Calculate the:
a) current in the edge MC if an ideal cell of emf E is connected across the adjacent corners A
and B.
b) current in the edge MA if an ideal cell of the emf E is connected across the opposite
corners A and C.
E E
Ans. a) b)
8R 2R

135. A hemispherical network of radius a is made by using a conducting wire of resistance per
unit length r. Find the equivalent resistance across OP.
CURRENT ELECTRICITY 104

ar
Ans. 2  
8

136. Find the equivalent resistance of the circuit between points A and B shown in figure is: (each
branch is of resistance  1 )

22
Ans. 
35

137. In the circuit shown in figure, all wires have equal resistance r. Find the equivalent
resistance between A and B.

3r
Ans.
5

138. A piece of resistive wire is made up into two squares with a common side of length 10 cm. A
currant enters the rectangular system at one of the corners and leaves at the diagonally
opposite corners. Show that the current in the common side is 1/5th of the entering current.
What length of wire connected between input and output terminals would have an equivalent
effect.
Ans. 7/5 times the length of any side of the square

139. The figure is made of a uniform wire and represents a regular five pointed star. The
resistance of a section EL is 2 ohm. Find the resistance of the star across F and C.

 1
 sin18 
 3
Ans. 2
105 CURRENT ELECTRICITY
140. The resistance of each resistor in the circuit diagram shown in figure is the same and equal
to R. The voltage across the terminals is U. Determine the current I in the leads if their
resistance can be neglected.

15 U
Ans. i
7 R

141. Three wires having resistance R a ,R b and R c are joined in the form of a triangle. Another wire
is joined at A and it makes a sliding contact with BC. The resistance of this wire is R d . Find
the maximum resistance of the circuit [assume all wires to be uniform]. (Arc BC is a part of a
circuit whose centre is at A)

 Ra   Ra 
 R b  2   Re  2  Rd
Ans.   
 Ra   Ra   R  R 
 R b  2  Rd   Rc  2    R b  a   Rc  a 
     2  2 
CURRENT ELECTRICITY 106

ELECTRICAL MEASURING INSTRUMENTS


Galvanometer
The most common instruments for measuring potential or current use a device called a
d’Arsonval galvanometer. A pivoted coil of fine wire is placed in the magnetic field of a
permanent magnet, as shown in figure. When there is a current in the coil, the magnetic field
exerts on the coil a torque that is proportional to the current. (This magnetic interaction is
discussed in detail in a later chapter.) This torque is opposed by a spring, similar to the
hairspring on the balance wheel of a watch, which exerts a restoring torque proportional to
the angular displacement.

Thus the angular deflection of the indicator needle attached to the pivoted coil is
directly proportional to the coil current, and the device can be calibrated to measure current.
The maximum deflection for which the meter is designed, typically 90 to 120 , is called full –
scale deflection. The current required to produce full –scale deflection (typically of the order
of 10 A to 10 mA ) and the resistance of the coil (typically of the order of 10 to 1000  ) are
the essential electrical characteristics of the meter.

The meter deflection is proportional to the current in the coil, but if the coil obeys
Ohm’s law, the current is proportional to the potential difference between the terminals of the
coil. Thus the deflection is also proportional to this potential difference. For example,
consider a meter whose coil has a resistance of 20  , and which deflects full scale with a
current of 1 mA in its coil. The corresponding potential difference is
Vab  IR  103 A   20    0.020 V  20 mV.

Ammeter
Let us consider further the use of the d’Arsonval meter as a current measuring
instrument, often called an ammeter. To measure the current in a circuit, an ammeter must
be inserted in series in the circuit so that the current to be measured actually passes
through the meter. If the galvanometer above is inserted in this way, it will measure any
current from zero to 1 mA. However, the resistance of the coil adds to the total resistance of
the circuit, with the result that the current after the galvanometer is inserted, although it is
107 CURRENT ELECTRICITY
correctly indicated by the instrument, may be less than it was before insertion of the
instrument. It is evidently desirable that the resistance of the instrument should be much
smaller than that of the remainder of the circuit, so that when the instrument is inserted it
does not change the very thing we wish to measure. An ideal ammeter would have zero
resistance.

Furthermore, the range of the galvanometer, if it is used without modification, is


limited to a maximum current of 1 mA. The range can be extended, and at the same time the
equivalent resistance can be reduced by connecting a low resistance S in parallel with the
moving coil, as in figure.

The parallel resistor is called a shunt; its effect is to permit some of the circuit current
I to bypass the meter and pass through the shunt instead. Usually the meter and the shunt
are mounted inside a case, with binding posts or plug connectors for external connections at
a and b. Occasionally, when several interchangeable shunts are used, they are mounted
outside the case.

SHUNT RESISTANCE TO CONVERT A GALVANOMETER INTO AN AMMETER OF DESIRED RANGE


If I is the range of the ammeter, Ig is the galvanometer current for full scale deflection,
G is the resistance of the coil and S is the shunt resistance then, clearly,
Ig G
 
I  Ig S  Ig G  S 
I  Ig

The resistance of the ammeter so obtained is


GS
RA  , generally, S  G , so G  S  G ,  R A  S
GS
i.e., ammeter is a low resistance device and hence it is always connected in series in a circuit.

Suppose we wish to modify the meter described above for use as an ammeter with a
range of 1 to 10 A. That is, the coil is to deflect full –scale when the current I in the circuit in
which the ammeter is inserted equals 10 A. The coil current Ig must then be 1 mA, so the
current I  Ig in the shunt is 9.999A. The potential difference Vab is Vab  Ig G   I  Ig  S.
CURRENT ELECTRICITY 108

 I   0.001 
Hence S  G  g   20    0.00200 .
II  9.999 
 g 
1 1 1
The equivalent resistance R of the instrument is   , and R A  0.00200 .
RA S G
Thus we have a low –resistance instrument with the desired range of 0 to 10 A. Of
course, if the current I is less than 10 A, the coil current (and the deflection) is
correspondingly less, also.

Note: i) The reading of an ammeter is always lesser than actual current in the circuit.

For example, in figure (a). actual current through R is,


E
i
R
 GS 
While the current after connecting an ammeter of resistance R A    in series with R is,
G  S
E
i' 
R  RA
From (i) and (ii), we see that i '  i and i '  i when R A  0 .
i.e., resistance of an ideal ammeter should be zero.
(ii) Percentage error in measuring a current through an ammeter is
1 1 
 i  i'  R  RR   RA 
A   100 or
   100   % error     100
 i   1   R  R 
   
A
 R 
Voltmeter
Now let us consider the construction of a voltmeter. A voltmeter measures the
potential difference between two points, and its terminals must be connected to these points.
A moving –coil meter cannot be used to measure the potential difference between, say two
charged spheres. When its terminals are connected to the spheres, the coil provides a
conducting path from one sphere to the other. There will be a momentary current in the coil,
but the charges on the sphere will change until the entire system is at the same potential.
Only if the resistance of the instrument is so great that a very long time is required to reach
equilibrium can a voltmeter be used for this purpose. Thus an ideal voltmeter has an infinite
resistance, but a moving –coil galvanometer can be deflected only by a current in its coil; its
resistance must be finite.
109 CURRENT ELECTRICITY
A moving – coil galvanometer can be used to measure the potential difference between
the terminals of a source, or between two points of a circuit containing a source, because the
source maintains a difference of potential between the points. However, the range of the
galvanometer in our example, if used without modification, is limited to a maximum value of
20 mV. The range can be extended, and at the same time the equivalent resistance can be
increased, by connecting a resistance R in series with the moving coil, as in figure.

SERIES RESISTANCE TO CONVERT A GALVANOMETER INTO A VOLTEMRERE OF DESIRED RANGE


If V is the desired range and R is the additional series resistance, then
V
V  Ig  G  R   R  G
Ig

Resistance of voltmeter R V  R  G . Voltmeter is a high resistance device and hence it is


always connected in parallel across a circuit whose voltage is to be measured.

A voltmeter, like an ammeter, can disturb the circuit to which it is connected. For
example, when a source is on open circuit, the potential difference between its terminals
equals its emf. It would seem, therefore, that to measure the emf we need only measure this
potential difference. But when the terminals of a voltmeter are connected to those of a
source, the meter and source form a complete circuit in which there is a current. The
potential difference after the meter is connected, although it is correctly indicated by the
instrument, is not equal to  , but to   Ir and is less than it was before the instrument was
connected again, the measuring instrument alters the quantity it is intended to measure. It is
evidently desirable that the resistance of a voltmeter, even though not infinite, should be as
large as possible.

Suppose we wish to modify the galvanometer for use as a voltmeter with a range of 0
to 10V. The coil is to deflect full – scale when the potential difference between the terminals
of the instrument is 10 V. In other words, the current in the instrument is to be 1 mA when
the potential difference between its terminals is 10 V.

The terminal potential difference is Vab  I  R  S , and the necessary series resistance is
Vab 10V
S R   20   9980  .
I 0.001A
CURRENT ELECTRICITY 110

The equivalent resistance R V is


R V  R  S  10,000  .
Thus we have a high –resistance instrument with the desired range of 0 to 10 V.

Note: i) The reading of a voltmeter is always lesser than true value.

For example, if a current i is passing through a resistance r, the actual value is,

V  ir ………………(i)
Now if a voltmeter of resistance R V   G  R  is connected across the resistance r, the new
value will be,
i   rR V  ir
V'  or V '  ………………(ii)
r  RV r
1
RV
From Eqs. (i) and (ii), we can see that,
V '  V and V '  V if R V  
Thus, resistance of an ideal voltmeter should be infinite,
(ii) Percentage error in measuring the potential difference by a voltmeter is,
   
 V  V'  1   1 
   100     100 or % error     100
 V  1  r  1  r 
     
 RV   RV 

A voltmeter and an ammeter can be used together to measure resistance and power.
The resistance of a resistor equals the potential difference Vab between its terminals, divided
Vab
by the current I: R  , and the power input to any portion of a circuit equals the product
I
of the potential difference across this portion and the current:
P  Vab I.

The most straightforward method of measuring R or P is therefore to measure Vab and I


simultaneously.

In figure (a) ammeter A reads correctly the current I in the resistor R. Voltmete V,
however, reads the sum of the potential difference Vab across the resistor and the potential
difference Vab across the ammeter.
111 CURRENT ELECTRICITY

If we transfer the voltmeter terminal from c to b, as in figure (b) the voltmeter reads
correctly the potential difference Vab but the ammeter now reads the sum of the current I in
the resistor and the current I in the voltmeter. Thus, whichever connection is used, we must
correct the reading of one instrument or the other to obtain the true values of Vab or I
(unless, of course, the corrections are small enough to be neglected).

LEARN THE SKILLS


74. What is the value of the shunt that passes 10% of the main current through a galvanometer
of 99  resistance.
Sol. Given G= 99  and Ig  10/100 I  0.1I

Ig G 0.1I  99 0.1
Now, S    99  11
 I  Ig   I  0.1 I  0.9

74. The deflection in a moving coil galvanometer falls from 50 divisions to 10 divisions when a
shunt of 12 is applied. What is the resistance of the galvanometer? Assume the main
current to remains same.
Sol. In case of a galvanometer, Ig   .
Ig10 1
Given, 

I 50 5
1
i.e., Ig  I
5
Now as in case of a shunted galvanometer as shown in figure.
 1  1
I  Ig  S  IgG or  I  I  12  Ig
 5  5
or G  4 12  48 

75. The resistance of the galvanometer G in the circuit is 25  . The meter deflects full scale for a
current of 10 mA. The meter behaves as an ammeter of three different ranges.
CURRENT ELECTRICITY 112

The range is 0-10 A, when the terminals O and P are taken. The range is 0 -1 A between O
and Q and the range is 0 -0.1 A between O and R. Calculate the resistance R1, R 2 and R 3 .
 I   102 
2  
Sol. Using, S   g  G  R1   25   0.025 
II  10  10 
 g 
Between O and Q, we have S  R1  R 2 .

 102 
2  
Hence, R1  R 2   25   0.2525   R 2  0.2275 
 1  10 
Between O and R, we get S  R1  R 2  R 3 .

 102 
2  
Hence, R1  R 2  R 3   25   2.78   R 3  2.5275 
 0.1  10 

76. A galvanometer has a resistance of 50  and its full-scale deflection current is 50 A . What
resistance should be added to it so that it can have a range of 0-5 V?
Sol. Given Ig  50 A . Maximum voltage to be measured is V=5 V. The galvanometer resistance
G  50  . Now,
V 5
R G   50  100 k
Ig 50  10 6

77. The scale of a galvanometer is divided into 150 equal divisions. The galvanometer has a
current sensitivity of 10 divisions per mA and a voltage sensitivity of 2 divisions per mV. How
can the galvanometer be designed to read (i) 6 A per division and (ii) 1 V per division?
Sol. Full-scale voltage= 150 division / 2divisions per mV =75 mV
Full-scale current = 150 divisions/10 divisions per mA =15 mA
Se resistance of galvanometer is

Full  scale voltage 75  103


G   5
Full  scale current 15  103

Ig G 15  103  5
(i) Range of ammeter is I  150  6  900 A . So, S    8.3  105 
I  Ig 900  15 10 
3

150
(ii) Range of voltmeter is V   5  9995 
15  10 3
113 CURRENT ELECTRICITY

78. A galvanometer has an internal resistance of 50  and current required for full scale
deflection is 1 mA. Find the series resistances required (as shown in figure) to use it as a
voltmeter with different ranges, as indicated in figure.

V
Sol. For range 1 V, galvanometer and R1 , are in series Ig 
 R1 
G 
1
 103   50  R1  1000  R1  1000  50  950 
50  R1 
For range 10 V, galvanometer and R 2 , R 3 are in series.
10 10
103   G  R1  R 2   10  103
 G  R1  R 2  10 3

 R2  1000  50  950   9000  R 2  9 k


and for range 100 V, galvanometer, R1, R2 and R3 are in series.
100 100
103   G  R1  R 2  R 3   100  103  R 3  100  103   G  R1  R 2 
 G  R1  R 2  R 3 10 3

 R3  100 103  10 103  R3  90 103  90k

79. A battery of emf 1.4 V and internal resistance 2 is connected to a resistor of 100  . In
order to measure the current through the resistance and the potential difference across its
ends, and ammeter is connected in series with it and a voltmeter is connected across its
4
ends. The resistance of the ammeter is  and that of the voltmeter is 200 . What are the
3
readings of the two instruments? What would be their reading if they were ideal instruments?
Sol. Let R A and R V be the resistances of the ammeter and voltmeter respectively. Then the total
RR V 100  200 4
resistance across the emf E is R eq   RA  r    2  70 
R  RV 100  200 3

E 1.4
Therefore, the current I0    0.02 A
R eq 70
This is the current through ammeter. Hence, the reading of ammeter is 0.02 A
CURRENT ELECTRICITY 114

Reading of voltmeter  potential difference across its terminals


 Reading of voltmeter
 RR V   100  200 
 I0    0.02    1.33 V
 R  RV   100  200 
If the ammeter and the voltmeter were ideal, R A  0 and R V  . Then,
E 1.4
The reading of ammeter    0.0137 A
r  R 2  100
1.4
The reading of voltmeter  I0 R   100  1.37 V
102

80. A voltmeter of resistance R1 and an ammeter of resistance R 2 are connected in series across

a battery of negligible internal resistance. When a resistance R is connected in parallel to


voltmeter, reading of ammeter increase three times while that of voltmeter reduces to one
third. Find R1 and R2 in terms of R.

Sol. Let E be the emf of the battery.

In the first case let i be the current in the circuit, then


E  i  R1  R2  ……………….(i)

In the second case main current increase three times while current through voltmeter will
reduce to i /3 . Hence, the remaining 3i  i /3  8i /3 passes through R as shown in figure.

i  8i  or R1  8R
VC  VD    R1    R
3 3
Applying Kirchhoff’s second laws in loop ABFGA
 R 
E  3i  R2    i /3  R1   i  3R 2  1  ……………..(ii)
 3 
R1 2R1 R 8R
From Eqs. (i) and (ii), R1  R 2  3R 2  or 2R 2  or R 2  1 or R 2 
3 3 3 3

81.
115 CURRENT ELECTRICITY

The circuit of figure is to be used to measure an un-known resistor R. The meter resistances
are R v  10,000  and R A  2.0  . If the voltmeter reads 12.0 V and the ammeter reads 0.10A
what is the true resistance?
Sol. If the meters were ideal (i.e., R v   and R A  0 ), the resistance would be simply
R  V /I  12.0 V  /  0.10 A   120  . But the voltmeter reading includes the potential
Vbc  IR A   0.10 A  2.0    0.2 V, so the actual potential drop Vab across the resistor is
Vab 11.8V
12.0 V  0.2V  11.8V, and the resistance is R    118 .
I 0.10 A

82. Suppose the same meters are connected instead as shown in figure and the above readings
are obtained. What is the true resistance?

Sol. In this case the voltmeter measures the potential across the resistor correctly; the difficulty is
that the ammeter measures the voltmeter current Iv as well as the current I in the resistor.

We have Iv  V /R v  12.0 V  / 10,000    1.2mA; so the actual current I in the resistor is

I  0.10 A  0.0012 A  0.0988A. Thus the resistance is

Vab 12.0 V
R   121.5 .
I 0.0988 A

83. An ammeter and a voltmeter are connected as in figures in order to measure an unknown
resistance. Justify quantitatively which connection is to be preferred.

Sol. Let  be the emf of the main source.


In Circuit-1:
CURRENT ELECTRICITY 116

  RV  X  R A 
i (main current)   .
RV  X  RA  RV  X  RA 
RV  X  RA
 RV  X  R A  RV  X  RA 
V ' (voltmeter reading ) = p.d. across the voltmeter    .
RV  X  RA  RV  X  RA
 RV  X  RA  
i' (ammeter reading) = Current in the branch   RV  .
R V  X  R A  R V  X  R A  X  RA
V'
X' (apparent value)   X  RA.
i'
 X ' X  RA
   % error      100   100.
 X  X
When X  0,    and hence this circuit is unsuitable for low resistance.
When X  ,   0 and hence this circuit is suitable for high resistance.
In circuit -2:
  RV  X 
i'  
 R A X R V  R A   R AR V
RV X
RV  X

RV X  RV  X  R V X
V'   
R V  X X  R V  R A   R AR V X R V  R A   R AR V

V' R V X X R A  R V   R AR V RV X
X'    
i' X  R V  R A   R A R V  R V  X  RV  X

 X ' X  X 100
   % error      100   100  .
 X  RV  X RV
1
X
When X  0,   0 and when X  ,   1.

Thus this circuit is suitable for low resistance.

INTEXT QUESTIONS
142. A galvanometer has a resistance of 30 ohm and a current of 2 mA is needed to give a full
scale deflection. What is the resistance needed to convert the galvanometer.
(a) Into an ammeter of 0.3 ampere range?
(b) Into a voltmeter of 0.2 volt range?
Ans. (a) 0.2013 (b) R  70 
143. A voltmeter of resistance 400 is used to measure the potential difference across the 100
resistor in the circuit shown in the figure. (a) What will be the reading of the voltmeter? (b)
What was the potential difference across 100 before the voltmeter was connected?
117 CURRENT ELECTRICITY

Ans. (a) 24 V (b) 28 V

144. A moving coil galvanometer of resistance 20  gives a full scale deflection when a current of 1
mA is passed through it. It is to be converted into an ammeter reading 20 A on full scale. But
the shunt of 0.005  only is available. What resistance should be connected in series with the
galvanometer coil?
Ans. R  79.995 

145. A resistance box, a battery and a galvanometer of resistance G  are connected in series. If
the galvanometer is shunted by resistance of S  , find the change in resistance in the box
required to maintain the current from the battery unchanged.
G2
Ans.
GS

146. A part of a circuit is shown in figure. Here reading of ammeter is 5 ampere and voltmeter is
96V & voltmeter resistance is 480 ohm. Then find the resistance R

Ans. 20 ohm

147. What resistance r should be used to shunt a galvanometer with an internal resistance of
R  10 k to reduce its sensitivity n  50 times?
Ans. 204 

148. What shunt resistance is required to make the 1 mA, 20  galvanometer into an ammeter
with a range of 0 A to 50 mA?
Ans. 0.408 

149. A voltmeter has a resistance G ohm and range V volt. Calculate the resistance to be used in
series with it to extend its range to nV volt.

Ans.  n  1 G
150. Assume that a galvanometer has an internal resistance of 60  and requires a current of 0.5
mA to produce full scale deflection. What resistance must be connected in parallel with the
galvanometer if the combination is to serve as an ammeter that has a full scale deflection for
a current of 0.1 A?
CURRENT ELECTRICITY 118

Ans. 0.302

151. Design a multi-range ammeter capable of full scale deflection for 25 mA, 50 mA, and 100 mA.
Assume galvanometer has a resistance of 25  and gives a full scale deflection for 1mA.
Ans.

R1  0.26 , R 2  0.261 , R 3  0.521 

152. Design a multi-range voltmeter capable of full scale deflection for 20 V, 50 V and 100 V,
Assume the galvanometer has a resistance of 60  and gives a full scale deflection for a
current of 1 mA.
Ans.

R1  19.94 k , R 2  30 k , R 3  50 k 

153 A galvanometer (coil resistance 99  ) is converted into a ammeter using a shunt of 1 and
connected as shown in the figure (i). The ammeter reads 3A. The same galvanometer is
converted into a voltmeter by connecting a resistance of 101 in series. This voltmeter is
connected as shown in figure (ii). Its reading is found to be 4/5 of the full scale reading. Find

(a) internal resistance r of the cell


(b) range of the ammeter and voltmeter
(c) full scale deflection current of the galvanometer
Ans. (a) 1.01W, (b) 0-5A, 0-10V, (c) 0.05A

154. A galvanometer having 50 divisions provided with a variable shunt s is used to measure the
current when connected in series with a resistance of 90  and a battery of internal
resistance10  . It is observed that when the shunt resistance are 10  , 50  , respectively
the deflection are respectively 9 & 30 divisions. What is the resistance of the galvanometer?
119 CURRENT ELECTRICITY
Further if the full scale deflection of the galvanometer movement is 300 mA, find the emf of
the cell.
Ans. 233.3; 144V

155. Draw the circuit for experimental verification of Ohm's law using a source of variable D.C.
voltage, a main resistance of 100  , two galvanometers and two resistances of values 106 
and 103  respectively. Clearly show the positions of the voltmeter and the ammeter.
Ans.

156. An ammeter and a voltmeter are connected in series to a battery with an emf   6.0V. When
a certain resistance is connected in parallel with the voltmeter, the reading of the voltmeter
decrease   2.0 times, whereas the reading of the ammeter increase the same number of
times. Find the voltmeter reading after the connection of the resistance.
Ans. 2.0 V

157. A battery of emf 1.4 V and internal resistance 2 is connected to a resistor of 100 through
an ammeter. The resistance of the ammeter is 4 /3 . A voltmeter has also been connected to
find the potential difference across the resistor.
(i) Draw the circuit diagram.
(ii) The ammeter reads 0.02 A. What is the resistance of the voltmeter?
(iii) The voltmeter reads 1.10 V, what is the zero error in the voltmeter?
Ans.

4
(i) (ii) 200  (iii) 1.1 –  0.23 V
3

158. A particular galvanometer serves as a 2 V full scale voltmeter when a 2500  resistor is
connected in series with it. It serves as a 0.5 A full scale ammeter when a 0.22  resistor is
connected in parallel with it. Determine the internal resistance of the galvanometer and the
current required to produce full scale deflection.
Ans. 145 , 0.756mA
159. Consider two different ammeters in which the currents are proportional to the respective
deflections of the needle. The first ammeter is connected to a resistor of resistance R1 and
the second to a resistor of unknown resistance R x . Firstly, the ammeters are connected in
series between points A and B (as shown in figure). In this case the readings of the ammeters
CURRENT ELECTRICITY 120

are n1 and n2 . Then the ammeters are connected in parallel between A and B (as shown in
figure) and indicate N1 and N2 . Determine the unknown resistance R x of the second resistor.

R1n2N1
Ans. Rx 
n1N2

160. A cell of emf 3.4 V and internal resistance 3  is connected to an ammeter having resistance
2 and to an external resistance of 100  resistance the ammeter reading is 0.04 A. Find
the voltage read by the voltmeter and its resistance. Had the voltmeter been an ideal one
what would have been its reading?
Ans. 3.2V, 3.238 V

161. In the circuit shown in figure, V1 and V2 are two voltmeters having resistances 6k and 4k
respectively. The emf of the battery is 250V, having negligible internal resistance. Two
resistances R1  4 k and R 2  6 k are also connected in the circuit as shown. Find the
reading of the voltmeters V1 and V2 when

(a) the switch S is open and (b) the switch S is closed.


Ans. (a) 150 V, 100 V (b) 125 V ,125 V

162. The emf E and the internal resistance r of the battery shown in figure are 4.3 V and 1
respectively. The external resistance R is 50  . The resistances of the ammeter and voltmeter
are 2 and 200  respectively.

(a) Find the readings of the two meters.


(b) The switch is thrown to the other side. What will be the readings of the two meters now?
Ans. (a) 4 V (b) 4.2 V, 0.08 A
121 CURRENT ELECTRICITY

163. Two resistor, 400  , and 800  are connected in series with a 6V battery. It is desired to
measure to the current in the circuit. An ammeter of 10  resistance is used for this
purpose. What will be the reading in the ammeter? Similarly, if a voltmeter of 1000 
resistance is used to measure the potential difference across the 400  resistor, what will be
the reading in the voltmeter?
Ans. 4.96 mA, 1.58 V

164. In figure, shown how to add just enough ammeters to measure every different current. Show
how to add just enough voltmeters to measure the potential difference across each resistor
and across each battery.

Ans.

165. A galvanometer having 30 divisions has current sensitivity of 20A /div. It has a resistance of
25 ohm. How will you convert it to an ammeter measuring upto1ampere? How will you now
convert this ammeter into a voltmeter reading upto 1 volt?
Ans.  0.015 in parallel; R  0.985  in series.

166. A potential difference of 220 volts is maintained across a 1200  rheostat as shown in fig.
The voltmeter V has a resistance of 600  and point c is at one fourth of the distance from a
to b. What is the reading of voltmeter?

Ans. 40 volt.

167. A galvanometer having a coil resistance of 100 ohms gives a full scale deflection when a
current of one milli-ampere is passed through it. What is the value of resistance which can
convert this galvanometer into ammeter giving a full scale deflection for a current of 10
amperes? A resistance of the required value is available but it will get burnt if the energy
CURRENT ELECTRICITY 122

dissipated in it is greater than one watt. Can it be used for the above described conversion of
the galvanometer? When this modified galvanometer is connected across the terminals of
battery, it shows a current 4 amp. The current drops to 1 amp., when the resistance of 1.5
ohm is connected in series with modified galvanometer. Find the emf and internal resistance
of battery.
Ans.  0.01, yes, 2V, 0.49.

STRETCHED-WIRE POTENTIOMETER
As ideal voltmeter which does not change the original potential difference, should have
infinite resistance. But in the design of voltmeter the resistance cannot be made infinite.
Potentiometer is a device which does not draw any current from the given circuit and still
measure the potential difference. Thus, it is equivalent to an ideal voltmeter.

The stretched-wire potentiometer consists of a long wire AB, usually 5 to 10 meters


long, fixed on a wooden platform. The wire has a uniform cross-section. Usually, separate
pieces of wire, each 1 m long, are fixed parallel to each other on the platform. The wires are
joined to each other by thick copper strips so that the combination acts as a single wire of
desired length (5 to 10 meters). The ends A and B are connected to a driving circuit
consisting of a strong battery, a plug key and a rheostat. The driving circuit sends a constant
current i through the wire AB. Thus, the potential gradually decreases from A to B. One end
of a galvanometer is connected to a metal rod fixed on the wooden platform. A “jockey” may
be slid on this metal rod may touch the wire AB at any desired point. In this way the
galvanometer gets connected to the point of AB which is touched by the jockey. The length of
the wire between the end A and this point can be measured with the help of a meter scale
fixed on the platform. The other end C of the galvanometer and the high-potential end A of
the wire, form the two end points (terminals) of the potentiometer. These points are
connected to the points between which the potential difference is to be measured.

Suppose, we have to measure the potential difference between the points a and b. Also
let a be at a higher potential and b at a lower potential. The end A of the wire AB is connected
to the point a and the end C of the galvanometer is connected to the point b. The circuit is
represented schematically in figure.

The connecting wire Aa has a negligible resistance and hence potentials of A and a are
equal. Suppose, the potential drop across ab is smaller than the potential drop across AB.
Then there will be a point P on AB which will have the same potential as b. If the jockey is
slid to touch the wire at this point P, the potential difference across the galvanometer is zero
and there will be no current through it. The process of measurement is to search for a point
P so that there is no deflection in the galvanometer.
123 CURRENT ELECTRICITY

Suppose, the driving circuit sets up a potential difference V0 between the ends A and
B of potentiometer wire. As the wire is uniform, the resistance of a piece of the wire is
proportional to its length. Hence, the potential difference across a piece of wire is also
proportional to its length. If AB=L and AP= , the potential difference between the points A
and P is

V  V0 …………..(i)
L
This is equal to the potential difference between a and b which we had to measure.
In order to get the value of the potential difference V, the total potential drop V0 on AB
must be known. One way to do this is to use a standard cell having known and constant emf
in place of ab. If the emf of the standard cell is  and the potentiometer is AP  0 , we have
from (i),

0 L
  V0 or V0  
L 0
The potential difference V between a and b is, from (i)
V .
0
This process of finding V0 is called calibration of the potentiometer. Note that there is
no current through the standard cell when the potentiometer is balanced during its
calibration. Thus, the emf  equals the potential difference between its terminals.

Comparison of Emf’s of Two Batteries

The driving circuit of the potentiometer is set up with a strong battery so that the
potential difference V0 across AB is larger than the emf of either battery. One of the batteries
is connected between the positive end A and the galvanometer. The jockey is adjusted to
touch the wire at a point P1 so that there is no deflection in the galvanometer. The length
AP1  1 is noted. Now, the first battery is replaced by the second and the length AP2  2 for
the balance is noted. If the length AB=L, the emf of the first battery is, from (i) above,

1  1V
0
L
and that of the second battery is
2
2  V0
L
1 1
Thus, 
2 2
CURRENT ELECTRICITY 124

Note that no calibration is needed in this case.

One can use a two-way key to connect both the batteries together as shown in figure.
When the key is pressed in the plug S1 , the first battery is brought into the circuit. When the
key is taken out from S1 and pressed in the plug S2 , the second battery is brought into the
circuit.
The value of emf of a battery can also the obtained by this same method by taking the
other battery to the a standard cell. The emf of the standard cell is known and hence the emf
of the given battery can be obtained.

Measurement of Internal Resistance of a Battery

Figure shows the arrangement for measuring the internal resistance of a battery. The
emf of the battery is  and its internal resistance is r. A known resistance R is connected
across the battery together with a plug key K. The potentiometer circuit is set up as usual.
The plug key K is opened and the balance point P is searched on the wire AB so that there is
no deflection in the galvanometer. As the key is open, there is no current through the
resistance R. Hence, there is no current through the battery and the potential difference
across the terminals a, b is the same as the emf  of the battery. If AP  , we have

 V0 ………………(i)
L
with the symbols having their usual meanings.
Now the key K is closed and the new balance point P ' is searched. There is a circuit.

i through the battery.
Rr
The potential difference between a and b is
R
Va  Vb  Ri  .
Rr

 ' V0
R
If AP '  ' , we have ………………(ii)
Rr L

Dividing (ii) by (i),


R
 ' or, r
R   '
Rr '
Potential Gradient (k)
Let 0 be the emf of the battery in the primary circuit and r its internal resistance. Let R h be
the resistance of the rheostat.
125 CURRENT ELECTRICITY
Potential difference (or fall in potential) per unit length of wire is called potential gradient
V0  0 
i.e., k where V0  IR   R
L  R  Rh  r 
V IR I 0 R
So, k 0   
L L A R  Rh  r  L
a) The potential gradient directly depends upon the
R
i) resistance per unit length   of potentiometer wire.
L
ii) radius of potentiometer wire (i.e., Area of cross section)
iii) specific resistance of the material of potentiometer wire (i.e.,  )
iv) current flowing through potentiometer wire (I)

b) Potential gradient indirectly depends upon the


i) emf of battery in the primary circuit (i.e., e)
ii) resistance of rheostat in the primary circuit (i.e., R h )

Sensitivity Of Potentiometer
A potentiometer is said to be more sensitive, if it measures a small potential difference more
accurately.
a) The sensitivity of potentiometer is assessed by its potential gradient. The sensitivity is
inversely proportional to the potential gradient.

b) In order to increase the sensitivity of potentiometer, the


i) resistance of primary circuit will have to be decreased.
ii) length of potentiometer wire will have to be increased so that the length may be measured
with more accuracy.

LEARN THE SKILLS


84. Figure shows a 2.0 V potentiometer used for the determination of internal resistance of a 1.5
V cell. The balance point of the cell in open circuit is 76.3 cm. When a resistor of 9.5 is
used in the external circuit of the cell, the balance point shifts to 64.8 cm length of the
potentiometer. Determine the internal resistance of the cell.

Sol. Here, 1  76.3cm, 2  64.8cm, r  ?, R  9.5 


CURRENT ELECTRICITY 126

 1  2   76.3  64.8 
Now, r   R   9.5  1.68 
 2   64.8 

85 Potentiometer wire PQ of 1 m length is connected to a standard cell E1 . Another cell E2 of


emf 1.02 V is connected with a resistance r and a switch S as shown in the circuit diagram.
With switch S open, the null position is obtained at a distance of 51 cm from P.
i) Calculate the potential gradient of the potentiometer wire.
ii) Find the emf of cell E.
iii) When switch S is closed, will the null point move toward P or toward Q? Given reason for
your answer.

V 1.02
Sol. i) Potential gradient is, k    2 Vm1
0.51
ii) The emf of the cell is, E1  k  2  1  2 V
iii) When the switch S is closed, there is no shift in the position of the null point because it
depends on the potential gradient along the potentiometer wire (which depends on the emf of
battery E1 and the resistance of the potentiometer wire circuit and length of
potentiometer) and the emf of cell E2 , which does not change when the switch S is closed.

86. In figure AB is a 1 m long uniform wire of 10  resistance. Other data are shown in the
figure. Calculate (i) potential gradient along AB and (ii) length AB and (ii) length AO when the
galvanometer shows no deflection.

Sol. i) Current in wire AB is I  2 15  10   2/25 A


Potential difference across AB is V  IR  2/25 10  0.8 V
Potential gradient along AB is k  V /  0.8/1  0.8 Vm1
1.5
ii) Current through 0.3  is  1A
1.2  0.3
Potential difference across 0.3  IS 1  0.3  0.3 V
Let 1 be the length AO, then 0.3  0.8  1 or 1  0.3 /0.8  0.375 m
127 CURRENT ELECTRICITY

87. A constant voltage V0 is applied to a potentiometer of resistance R connected to an ammeter.


A constant resistor r is connected to the sliding contact of the potentiometer and the fixed
end of the potentiometer. How will the reading of ammeter vary as the sliding contact is
moved from one end of the potentiometer to the other. The resistance of ammeter is assumed
to be negligible.

Sol. If x is the resistance of the potentiometer between point a and the sliding contact, the total
rx
resistance between a and the sliding contact is , while the resistance of the entire
rx
rx V0
circuit is  R  x   . The current supplied by the source is I 
rx rx
Rx
rx
The potential difference between the sliding contact and point a is
V0 rx V0 rx
V 
 R  x  r  x   rx Rx  x 2  Rr
V0 r
The current passing through ammeter is, Ia 
Rx  x 2  Rr
dIa
To find the extremum (maximum, minimum or constant) 0
dx
 
R  2x   0 x  R
 V0 r 
  Rx  x  Rr  
2 2
2
 
V0 r
Substituting x in equation (1) we find the minimum current Imin 
 R
R r  
 4
Thus, as the sliding contact is moved the current through the ammeter passes through a
minimum and the smaller the r the deeper the minimum. At x  0 and x  R, a current of
V0 I x r
Imax  passes through the ammeter. The a versus curves for several values of are
R Imax R R
shown in figure.
CURRENT ELECTRICITY 128

The Metre Bridge


The meter bridge is the practical application of the wheat stone network principle in which
the ratio of two of the resistances, say R and S, is deduced from the ratio of their balancing
lengths. AC is a 1 m long uniform wire. If AD  cm, then DC  100   cm since Resistance
 Length

P
 
Q 100 
If P is known then Q can be determined.
At the balancing point galvanometer G gives no deflection at all. At A and C, the
galvanometer must have deflections in opposite direction because, then only zero deflection
can be expected when the jockey (J) attached to the galvanometer is moved from A to C.

The Post Office Box


It is a compact form of the wheat-stone bridge. It consists of compact resistance so arranged
that different desired values of resistances may be selected in the three arms of wheat-stone
bridge, as shown in figure.

Each of the arms AB and BC contains three resistances of 10,102 and 103 , respectively.
R2
These are called the ratio arms. Using these resistances the ratio can be made to have
R1
any of the following values: 100: 1, 10: 1, 1: 1, 1: 10 or 1: 100.
129 CURRENT ELECTRICITY
The arm AD is complete resistance box containing resistance from 1 to 5000 . The tap keys
K1 and K 2 are also provided in the post office box. The key K1 is internally connected to the
point A and the key K 2 to the point B (as shown by dotted line in the figure.) The unknown
resistance X is connected between C and D, the battery between C and the key K1 and the
galvanometer between D and the key K 2 . The circuit shown in figure (A) is exactly the same
as that of the wheat-stone bridge shown in figure (B).
R 
Hence, the value of the unknown resistance is given by X  R  2 
 R1 
Note:
R2
(i) the accuracy of the post office box depends on the choice of ratio arm .
R1
(ii) If R 2 :R1 is 1 : 1, then the value of the unknown resistance is obtained within 1 .
 1 
(iii) If the ratio R 2 :R1 is selected as 1 : 10, then the unknown resistance X  R   is
 10 
accurately measured upto 0.1.
(iv) If the ratio R 2 :R1 is adjusted to 1 : 100, then the value of unknown resistance
 1 
X  R  is obtained to an accuracy of 0.01.
 100 
CURRENT ELECTRICITY 130

LEARN THE SKILLS


88. In the simple potentiometer circuit, where the length AB of the potentiometer wire is 1m, the
resistors X and Y have values of 5  and 2  , respectively. When X is shunted by a wire, the
balance point is found to be 0.625 m from A. What is the resistance of the shunt?

Sol. Let R be the resistance of the shunted wire. The effective resistance of R and 5  in parallel
is 5R /  5  R  .
5R /  5  R  0.625 0.625 5
At balance point,    , on solving, we get R  10  .
2 1  0.625 0.0375 3

INTEXT QUESTIONS
168. In a potentiometer experiment it is found that no current passes through the galvanometer
when the terminals of the cell are connected across 0.52 m of the potentiometer wire. If the
cell is shunted by a resistance of 5  a balance is obtained when the cell is connected across
0.4 m of the wire. Find the internal resistance of the cell.
Ans. 1.5 

169. How a battery is to be connected so that shown rheostat will behave like a potential divider?
Also indicate the points about which output can be taken.

Ans. Battery should be connected across A and B. Output can be taken across the terminals A
and C or B and C

170. An ideal cell having a steady emf of 2 volt is connected across the potentiometer wire of
length 10 m. The potentiometer wire is of magnesium and having resistance of 11.5 /m. An
another cell gives a null point at 6.9 m. If a resistance of 5 is put in series with
potentiometer wire, find the new position of the null point A.
Ans. 7.2m

171. In the figure shown for given values of R1 and R 2 the balance point for Jockey is at 40 cm
from A. When R 2 is shunted by a resistance of 10 , balance shifts to 50 cm. find R1 and R 2 .
(AB = 1 m):

10
Ans. ,5
3
131 CURRENT ELECTRICITY
172. A 6 volt battery of negligible internal resistance is connected across a uniform wire AB of
length 100 cm. The positive terminal of another battery of emf 4V and internal resistance 1
 is joined to the point A as shown in figure. Take the potential at B to be zero. (a) What are
the potentials at the points A and C? (b) At which point D of the wire AB, the potential is
equal to the potential at C. (c) If the point C and D are connected by a wire, what will be the
current through it? (d) If the 4V battery is replaced by 7.5 V battery, what would be the
answers of parts (a) and (b)?

200
Ans. (a) 6 V, 2 V (b) AD   66.7cm (c) zero (d) 6 V, – 1.5 V, no such point D exists.
3

173. The emf e and the internal resistance r of the battery shown in figure are 4.3 V and 1.0 
respectively. The external resistance R is 50  . The resistances of the ammeter and voltmeter
are 2.0  and 200  respectively. (a) Find the readings of the two meters. (b) The switch is
thrown to the other side. What will be the readings of the two meters now?

Ans. (a) 0.1A, 4.0 V (b) 0.08A, 4.2V

174. Figure shows a 2.0 V potentiometer used for the determination of internal resistance of 1.5 V
cell. The balance point of the cell without 9.5 in the external circuit is 70 cm. When a
resistor of 9.5 is used in the external circuit of the cell, the balance point shifts to 60 cm
length of the potentiometer wire. Determine the internal resistance of the secondary cell.

9.5
Ans. ohm
6

175. Figure shows a potentiometer with a cell of emf 2.0 V and internal resistance 0.04
maintaining a potential drop across the potentiometer wire AB. A standard cell which
maintains a constant emf of 1.02 V (for very moderate currents up to a few ampere) gives a
balance point of 67.3 cm length of the wire. To ensure very low currents drawn from the
standard cell, a very high resistance of 600k is put in series with it which is shorted close
to the balance point. The standard cell is then replaced by a cell of unknown emf E and the
balance point found similarly turns out to be at 82.3 cm length of the wire.
CURRENT ELECTRICITY 132

(a) What is the value of E?


(b) What purpose does the high resistance of 600k have?
(c) Is the balance point affected by this high resistance?
(d) Is the balance point affected by the internal resistance of the driver cell?
(e) Would the method work in the above situation if the driver cell of the potentiometer had
an emf of 1.0 V instead of 2.0 V?
(f) Would the circuit work well for determining extremely small emf, say, of the order of few
mV (such typical emf of thermocouple)?
Ans. (a) 1.25 V, (b) saving of galvanometer from damage and to prevent the cell
discharging fast
(c) No (d) Yes (e) No (f) No

176. A battery of emf 0  10V is connected across a 1 m long uniform wire having resistance
10 /m. Two cells of emf 1  2V and 2  4V having internal resistances 1 and 5
respectively are connected as shown in the figure. If a galvanometer shows no deflection at
the point P, find the distance of point P from the point a.

Ans. 46.67 cm

177. A potentiometer wire AB is 100 cm long and has a total resistance of 10 ohm. If the
galvanometer shows zero deflection at the position C, then find the value of unknown
resistance R.

Ans. 4 ohm
133 CURRENT ELECTRICITY
178. In the primary circuit of potentiometer the rheostat can be varied from 0 to 10  . Initially it is
at minimum resistance (zero).
(a) Find the length AP of the wire such that the galvanometer shows zero deflection.

(b) Now the rheostat is put at maximum resistance ( 10  ) and the switch S is closed. New
balancing length is found to 8m. Find the internal resistance r of the 4.5V cell.
Ans. (a) 6m, (b) 1

179.. Figure shows a metre bridge (which is nothing but a practical Wheatstone Bridge) consisting
of two resistors X and Y together in parallel with a metre long constantan wire of uniform
cross-section. With the help of a movable contact D, one can change the ratio of the
resistances of the two segments of the wire until a sensitive galvanometer G connected across
B and D shows no deflection. The null point is found to be at a distance of 30 cm from the
end A. The resistor Y is shunted by a resistance of 12.0 and the null point is found to shift
by a distance of 10 cm. Determine the resistance of X and Y.

20 20
Ans. X , Y  
7 3

180. A thin uniform wire AB of length 1 m, an unknown resistance X and a resistance of 12  are
connected by thick conducting strips, as shown in the figure. A battery and a galvanometer
(with a sliding jockey connected to it) are also available. Connections are to be made to
measure the unknown resistance X using the principle of Wheatstone bridge. Answer the
following question.

(a) Are there positive and negative terminals on the galvanometer?


(b) Copy the figure in your answer book and show the battery and the galvanometer (with
jockey) connected at appropriate points.
(c) After appropriate connections are made, it is found that no deflection takes place in the
galvanometer when the sliding jockey touches the wire at a distance of 60 cm from A. Obtain
the value of the resistance X.
Ans.
CURRENT ELECTRICITY 134

(a) No (b) or (c) 8 

181. In a potentiometer circuit, two wires of same material of resistivity  , one of radius of cross-
section ' a ' and other of radius of cross-section ' 2 a ' are joined in series. They are of length
and 2 respectively. This combination acts as the potentiometer wire of length 3 . The emf

of the cell in the primary circuit is e and internal resistance is . This cell is connected to
2a 2
the potentiometer wire by a conducting wire of negligible resistance with positive terminal of
the cell connected to one end (call it A) of longer wire. The negative terminal of the cell is
connected to one end of the smaller wire. The remaining ends of the two wires are joined
together. Find:
(i) The maximum voltage which can be balanced on the potentiometer wire.

(ii) The length, measured from point A, where cell of emf will balance.
2
 
(iii) If positive terminal of cell of emf
and internal resistance is connected to point A
2 2a 2
and other terminal is joined to the junction of the two wires, then find the current through
this cell.
3 5  
Ans. (i) v 0  (ii) (iii) , where R  and A  2a 2
4 2 7R A

HEATING EFFECT OF CURRENT


When a charge dq passes across a potential difference V, the work done dW is given by,
dW  Vdq
This work represents the loss of potential energy of charges. The flow of charge dq in time dt
dq
is equivalent to current I, where I 
dt
 dq  Idt  dW  VIdt

If constant current I passes through conductor for time t under a potential difference V, then
W  VIt …….(1)
According to Ohm’s Law, V  IR
2
 Work done  I2Rt  V t  VIt …….(2)
R
This work is converted into the energy of random thermal motion of molecules of the
conductor. That is the electric current through a conductor produces thermal energy in the
conductor and the conductor gets heated. This phenomenon is called Joule’s heating effect of
current.
135 CURRENT ELECTRICITY

V2
If V is in volt, I in amp, R in ohm, then Joule’s heat is equivalent to W  VIt  I2Rt  t joule
R
 V2 
 t
W VIt I2Rt  R 
or Heat produced Q is given by Q     ca
J J J J

Power
The rate of work done in an electrical circuit is called the power and is dissipated in the form
dW V2
of heat. Power dissipated, P   VI  I2R 
dt R
The SI unit of power is watt(W) 1 W   1Js 1

Units Of Electric Energy And Electric Power


Electric energy can be expressed in units such as J, cal, k Wh, etc.
1cal  4.18 J  4.2 J

Relation Between k Wh and J:

1kWh  1000 W  h  1000 W  3600s  3.6 106 Ws  3.6 106 J


1 kWh is the energy consumed by an appliance of power 1 kW when it runs for 1 h.

Commercial Unit:
1 kWh is one unit of electricity. To calculate the number of units, we can use the following
relation:
watt  hour
Number of units 
1000
The energy dissipated in k Wh can be calculated using the following relation:

V  in volt   I  in ampere   t  in hour 


E
1000
Electric power can be expressed in units such as W, kW, MW, hp; they share the following
relationship: 1kW  103 W,1MW  106 W,1hp  746W .

Note:
1) If the resistances are connected in series, then using, P  I2 R , the power developed will be
higher in the resistor of higher value as current will be same in all resistors.
2) If the resistances are connected in parallel, then using P  V2 /R , the power developed will
be higher in the resistor of lower value as potential will be same across all resistors.

Specification Of A Bulb Or Other Electric Appliances


Let a bulb be designed to operate on a voltage V0 and its power indicated on it be P0 . The
resistance of the bulb is given by R  V02 / P0 . Now let a potential difference of V is applied
across this bulb, then power consumed is given by
CURRENT ELECTRICITY 136

2
V2  V 
P   P0
R  V0 
If V  V0 , then P  P0 . The above formula is very convenient to calculate the power
consumption when the applied voltage is different from the specified one.

An electric appliance consumes the specified power P0 only if it runs at the specified
voltage V0 . If the applied voltage VA is greater than the specified voltage, the appliance may
get damaged as in this situation, I  VA / R will exceed its current capacity IC  V0 /R .
Further, if an appliance in made to run at a voltage lower than the specified, then true power
consumption will be less the specified value.

POWER TRANSFORMATION RULE


A) When Bulbs (Or Electrical Appliances) Are Connected In Parallel.
Let R1, R 2 , R 3 ...... be resistance of given bulbs meant to operate at same voltage V to
consume powers P1, P2 , P3 ,.......
V2 V2 V2 V2 V2 V2
Then R1  , R2  , R3  , .......  P1  ,P2  ,P3  , ......
P1 P2 P3 R1 R2 R3
1 1 1 1
When connected in parallel, their combined resistance R is     ........
R R1 R2 R3

 Power consumed P  V  V 2  1  1  1  ...... 


2

R  R1 R2 R3 
2 2 2
V V V
P    .....  P  P1  P2  P3  .....
R1 R 2 R 3

B) When Bulbs (Or Electrical Appliances) Are Connected In Series Across V Mains
Net resistance in series R  R1  R 2  R 3  ......
 Net power consumed is given by P  I2R1  I2R2  I2R3  ......
 P  P1  P2  P3  .......

Maximum Power Transfer Theorem


The power consumption across the external resistance of the circuit is maximum when the
net external resistance of the circuit equals the net internal resistance.
E
Consider the circuit shown. The power consumed across R is I2R, Where I  .
Rr
137 CURRENT ELECTRICITY

E 2R E 2R
P P
R  r  R  r 
2 2
 4Rr
For P to be maximum, R  r  0 Rr
So, we observe that for maximum power consumption across the external resistance, we
must have External Resistance = Internal Resistance of the battery.
E2 E2
 Pmax  
4R 4r
This is an example of “impedance matching”, in which the variable resistance R is adjusted
so that the power delivered to it is maximized. The behaviors of P as a function of R is
depicted in figure below.

Characteristics Of Fuse
Fuse is used with the main electrical circuit for the safety of electrical appliances.
A fuse wire must have high resistance and low melting point. Hence, generally it is made of
tin - lead alloy.
Let R be the resistance,  resistivity, length, A cross-sectional area and I be its current
carrying capacity. When the fuse is safe, then for its steady state temperature, heat produced
per second must be equal to heat radiated by it per second. Heat produced in fuse wire per
Q 2  
second H   I R  I2  
t A
I2
H joule sec 1 ……..(1)
r2
If e is emissivity of fuse material of radius r and T is the excess safe temperature of wire
above surrounding then according to Newton’s Law of Cooling, the energy radiated per second
H  e  2r  T …… (2)

For steady state e  2r  T  I2
r 2
I2
 T …….(3)
22er3
So, we observe that the steady state temperature of a fuse is independent of its length.
Hence length is immaterial for an electric fuse.
For a given material of a fuse wire I2  r3
CURRENT ELECTRICITY 138

LEARN THE SKILLS


89. Two wires of same mass, having ratio of lengths 1:2, density 1:3, and resistivity 2:1, are
connected one by one to the same voltage supply. The rate of heat dissipation in the first wire
is found to be 10 W. Find the rate of heat dissipation in the second wire.
Sol. Given
1  1 , d1  1 , 1  2 and m  A
1 1d1  A2 2d2
2 2 d2 3 2 1
P2 V 2 /R 2 R  A  2d 2 1
2
1 1 P 10 5
 2  1  1 1 2  1 12 1       or P2  1   W
P1 V /R1 R2 2 2A1 2 2d2 1 2 3 6 6 6 3

90. A 100 W bulb is designed to operate on a potential difference of 230 V.


i) Find the resistance of the bulb.
ii) Find the current drawn by the bulb if it is operated at a potential difference for which it is
designed.
iii) Find the current drawn and power consumed by the bulb if it is connected to a 200 V
supply.
Sol. Power rating of the bulb is P0  100 W , voltage and rating of the bulb is V0  230 V
2
V 2  230 
i) Resistance of the bulb is R  0   529 
P0 100
V0 230 10
ii) Current drawn is I    A
R 529 23
2
V 200  200 
iii) I   A  P  I2R     529  75.6 W
R 529  529 

91. A 500 W heating unit is designed to operate from a 200 V line. By what percentage will its
heat output drop if the line voltage drops to 160 V? Find the heat produced by it in 10 min.
Sol. Actual power consumed is
2
 V   160 
P  P0    500  320 W
 V0   200 
Heat output drop is 500  320  180 W
180
Percentage heat drop is  100  36%
500
Thus, heat produced in 10 min (600 s) is given by
H  320  600  192000 J  192 kJ

92. A battery has an open circuit potential difference of 6 V between its terminals. When a load
resistance of 60  is connected across the battery, the total power dissipated by the battery is
0.4 W. What should be the load resistance R, so that maximum power will be dissipated in R.
Calculate this power. What is the total power supplied by the battery when such a load is
connected?
Sol. When the circuit is open, V  E
139 CURRENT ELECTRICITY
 E  6 volt

Let r be the internal resistance of the battery.


Power supplied by the battery in this case is,
E2
P
Rr
 6 2
Substituting the values, we have 0.4 
60  r
Solving this, we get r  30 
Maximum power is dissipated in the circuit when, net external resistance is equal to
net internal resistance or,
R  r  R  30 
Further, total power supplied by the battery under this condition is,
2
PTotal 
E2

 6   0.6 watt
R  r 30  30
Of this 0.6 W half of the power is dissipated in R and half in r. Therefore, maximum power
0.6
dissipated in R would be  0.3 watt
2

93. In which branch of the circuit shown in figure a 11 V battery be inserted so that it dissipates
minimum power. What will be the current through the 2 resistance for this position of the
battery?

Sol. Suppose, we insert the battery with 2 resistance. Then we can take 2 as the internal
resistance (r) of the battery and combined resistance of the other two as the external
resistance (R). The circuit in that case shown in figure (a).

Now power,
CURRENT ELECTRICITY 140

E2
P
Rr
This power will be minimum where R  r is maximum and we can see that  R  r  will be
maximum when the battery is inserted with 6  resistance as shown in figure (b).
Net resistance in this case is
2  4 22 11
6    i  1.5 A
24 3 22/3
This current will be distributed in 2 and 4  in the inverse ration of their resistance.
i1 4  2 
   2 i1    1.5   1.0 A
i2 2  2 1

94. A circuit shown in the figure has resistance 20  and 30  . At what value of resistance R x
will the thermal power generated in it be practically independent of small variations of that
resistance? The voltage between points A and B is supposed to be constant in this case.

Sol. V = Constant
Now, voltage across R x is equal to the voltage across the 30 resistor, because both are in
parallel. Hence, if V1 is the voltage across R x , then
 30R x 
 30  R   30 R x 
V1  V30   x V   V
 30R x  20   50R x  600 
 30  R 
 x 

V12 900R x V 2
Now, power generated in R x is P  
R x  50R x  600 2
dP
For P to be constant, we have 0
dR x


 50R x  600 
2
 900V   1800 50  R
2
x V 2  50R x  600 
0
 50R x  600 
4

 50R x  600  100R x  0  R x  12 


95. Two bulbs are rated 30 W- 200 V and 60 W- 200V. They are connected with a 400 V power
supply. Find which bulb will get fused if they are connected in (i) series and (ii) parallel.
141 CURRENT ELECTRICITY
2 2

 200 
, R 
 200 
  R1  R 2
Sol. i) R1 2
30 60

Hence, voltage across the first bulb will be greater than 200 V. So it will get fused.
ii) In parallel, potential across both the bulbs will be same and equal to 400 V. So both will
get fused.

96. How will you connect (series and parallel) 24 cells each of internal resistance 1 to get
maximum power output across a load of 10  ?
Sol. Suppose there are m rows and each row has n cells. The total number of cells is mn=24. Here
nE
I
R   nr / m 
For maximum power
nr
 R or n  10m or10 m2  24 or m  2.4  1.55
m
2
24 E  24E 
i) If m=1, n=24, then I  , then P1     10  4.98 E2
34  34 
2
12 E  12E 
ii) If m=2, n=12, then I  , then P2     10  5.625 E2
16  16 
So we have two rows (m=2) each containing 12 cells (n=12) in series.

97. What amount of heat will be generated in a coil of resistance R due to a charge q passing
through it if current in the coil.
a) decreases down to zero uniformly during a time interval t 0 ?
b) decreases down to zero halving its value every t 0 seconds?
How to proceed
Heat generated in a resistance is given by,
H  i2Rt
We can directly use this formula provided i is constant. Here, i is varying. So, first we will
calculate i at any time t, then find a small heat dH in a short interval of time dt. Then by
integrating it with proper limits we can obtain the total heat produced.
Sol. (a) The corresponding i  t graph will be a straight line with i decreasing from a peak value
(say i0 ) to zero in time t0 .
i  t equation will be as
i 
i  i0   0  t  y  mx  c  ……….(i)
 t0 
CURRENT ELECTRICITY 142

Here, i0 is unknown, which can be obtained by using the fact that area under i-t graph gives
the flow of charge. Hence,
1
q   t0  i0   i0  2q
2 t0
2q  t   2q 2qt 
Substituting in (i), we get, i  1   or i  2 
t0  t0   t0 t0 
2
 2q 2qt 
Now at time t, heat produced in a short interval dt is, dH  i2R dt    2  Rdt
 t0 t0 
2
t0 t0  2q 2qt  4 q2 R
 Total heat produced   dH or H     2  R dt 
0 0
 t0 t0  3 t0
(b) Here, current decreases from some peak value (say i0 ) to zero exponentially with half life
t 0 . i  t equation in this case will be i  i0et
n  2
Here, 
t0
i0 
Now, q   i dt   i0etdt  
 

0 0


 i 0  q
 i   q  et
  q 2 R
 dH  i2R dt  2q 2e2t R dt or H   dH  2q 2R  e2t dt 
0 0 2
n  2 q 2 R n  2
Substituting   , we have H  .
t0 2t 0

98. A conductor has a temperature independent R and a total heat capacity C. At the moment
t  0 it is connected to a dc voltage V. Find the time dependence of the conductor’s
temperature T assuming the thermal power dissipated into surrounding space to vary as
q  k  T  T0  where k is a constant, T0 is the environmental temperature (equal to

conductor’s temperature at the initial moment).


143 CURRENT ELECTRICITY

V2
HOW TO PROCEED Here energy is being generated in the resistance at a rate of . Of
R
which, part of energy is being lost in the environment and the rest is utilized in raising the
temperature of conductor. So, this is basically a problem of conservation of energy.
Sol. Energy supplied by the dc source per unit time = energy lost in environment per unit time +
energy used in raising the temperature of conductor per unit time.
V2  dT    dT  V
2
dT dt
Hence,  k  T  T0   C   C     k  T  T0   2 
R  dt   dt  R V C
 k  T  T0 
R
T dT t dt
Substituting the proper limits, we have T0 V2
 
0 C
 k  T  T0 
R

V2  
kt

Solving this equation, we get T  T0  1  e C 
kR  

INTEXT QUESTIONS

182. Two resistors are made of identical material, have the same length but one has the diameter
twice that of the other. Determine the ratio of powers dissipated in the two resistors when
(a) each resistor is being connected across the same voltage source i.e. parallel
(b) they are connected in series
1
Ans. (a) 4 (b)
4

183. A 100 W, 110 V light bulb has a filament made of an alloy having a temperature coefficient of
0.0055 C1 at 0 C. The normal operating temperature of the bulb is 2000 C. How much
current will the bulb draw at the instant it is turned on when the room temperature is 20 C
and 2000 C ?
Ans. I20  9.82 A ; I2000  0.91A

184. In the circuit shown in figure, find the power

(a) supplied by 10 V battery (b) consumed by 4 V battery and (c) dissipated in 3 


resistance.
Ans. (a) 20 W (b) 8 W (c) 12 W
CURRENT ELECTRICITY 144

185. Calculate the power delivered to each resistor in the circuit shown.

Ans. 14.2 W in 2 , 1.33 W in 3  , 4 W in 1

186. In following diagram boxes may contain resistor or battery or any other element

then determine in each case


(a) E.m.f. of battery
(b) Battery is acting as a source or load
(c) Potential difference across each battery
(d) Power input to the battery or output by the battery.
(e) The rate at which heat is generated inside the battery.
(f) The rate at which the chemical energy of the cell is consumed or increased.
(g) Potential difference across box
(h) Electric power output across box.
Ans. (a) E = 10 V each (b) (A) act as a source and (B) act as load
(c) VA  9V,VB  11V (d) PA  9W,PB  11W
(e) Heat rate = 1 W each (f) 10 W each
(g) 9V, 11V (h) –9W, 11 W

187. A resistor with a current of 3 A through it converts 500 J of electrical energy to heat energy
in 12 s. What is the voltage across the resistor?
125
Ans. V
9

188. The figure shows the current I in a single-loop circuit with a battery B and resistance R (and
wires of negligible resistance). Then find the order of following at the point a,b and c

(a) The magnitude of the current,


(b) The electric potential, and
(c) The electric potential energy of the charge carriers (electron), greatest first.
Ans. (a) all equal (b) b, then a and c equal (c) a and c equal, b
189. 1 kW, 220 V electric heater is to be used with 220 V D.C. supply.
145 CURRENT ELECTRICITY
(a) What is the current in the heater?
(b) What is its resistance?
(c) What is the power dissipated in the heater.
(d) How much heat in calories is produced per second?
(e) How many grams of water at 100C will be converted per minute into steam at 100C
with the heater. (Latent heat of vaporization of water  540ca /g )] [ J  4.2 J /ca ]
Ans. (a) 4.55A (b) 48.4 (c) 1000 W (d) 240ca s 1 (e) 80/3 gm

190. Two electric bulbs, each designed to operate with a power of 500 watts in 220 volt line, are
connected in series. Now they are connected with a 110 volt line. What will be the power
generated by each bulb?
Ans. 31.25 watt

191. A person decides to use his bath tub water to generate electric power to run a 40 watt bulb.
The bath tube is located at a height of 10 m from the ground & it holds 200 litres of water. If
he install a water driven wheel generator on the ground, at what rate should the water drain
from the bath tube to light bulb? How long can we keep the bulb on, if the bath tub was full
initially? The efficiency of generator is 90%.  g  10m /s 2 
Ans. 4/9 kg/sec., 450 sec

192. It is required to send a current of 8 A through a circuit whose resistance is 5  . What is the
least number of accumulators which must be used for this purpose and how should they by
connected? The emf of each accumulator is 2 V and the internal resistance is 0.5 .
Ans. 160, Four rows containing 40 cells in each row.

193. Compare the brightness of four identical light bulbs in figure. What happens if bulb A fails so
that it cannot conduct? What if C fails? What if D fails?

Ans. Brightness of C is the greatest and that of A and B are equal.


If A fails, B will not glow and no effect on C and D.
If C fails, C will not glow and no effect on A, B, and D.
If D fails, no effect anywhere.

194. A series circuit consists of three identical lamps connected to a battery as shown in figure.

When the switch S is closed, what happens


a) to the intensities of lamps A and B,
CURRENT ELECTRICITY 146

b) to the intensity of lamp C,


c) to the current in the circuit, and
d) to the voltage drop across the three lamps?
Does the power dissipated in the circuit increase, decrease, or remain the same?
Ans. a) Will increase and become 2.25 of their initial values.
b) Will become zero.
c) Will increase and will become 1.5 times its initial value.
d) Increases and becomes 1.5 times its initial value.

195. An electric teakettle has a multi-position switch and two heating coils. When only one of the
coils is switched on, the well –insulated kettle makes a full pot of water to a boil in a time
interval t. When only the other coil is switched on, it requires a time interval of 2t to boil
the same amount of water. Find the time interval required to boil the same amount of water if
both coils are switched while being used
(a) in a parallel connection and (b) in a series connection.
2 t
Ans. (a) (b) 3t
3
196. When two unknown resistors are connected in series with a battery, the battery delivers total
power Ps and carries a total current of I. For the same total current, a total power Pp is

delivered when the resistors are connected in parallel. Determine the values of the two
resistors.

Ps  Ps2  4Ps Pp Ps  Ps2  4Ps Pp


Ans. ,
2I2 2I2

197. In a circuit shown in figure if the internal resistance of the sources are negligible then at
what value of resistance R will the thermal power generated in it be the maximum. What is
its value?

Ans. 2 , 4.5 W

198. In the circuit shown in figure, the emfs of batteries are E1 and E2 which have internal
resistance R1 and R 2 . At what value of the resistance R will the thermal power generated in it
be the highest? What it is?
147 CURRENT ELECTRICITY

 E1R2  E2R1 
2
2R1R 2
Ans. ,
R1  R 2 4R1R 2  R1  R 2 

199. If a cell of constant E.M.F. produces the same amount of the heat during the same time in
two independent resistors R1 and R 2 , when they are separately connected across the terminals
of the cell, one after the another, find the internal resistance of the cell.
Ans. R1R 2

200. Find the resistor in which maximum heat will be produced.

Ans. 4

201. Three equal resistance each of R ohm are connected as shown in figure. A battery of 2 volts
of internal resistance 0.1 ohm is connected across the circuit. Calculate R for which the heat
generated in the circuit is maximum.

Ans. 0.3

202. The current I through a rod of a certain metallic oxide is given by I  0.2V 5/2 , where V is the
potential difference across it. The rod is connected in series with a resistance to a 6V battery
of negligible internal resistance. What value should the series resistance have so that:
(i) the current in the circuit is 0.44
(ii) the power dissipated in the rod is twice that dissipated in the resistance.
Ans. (i) 10.52 (ii) 0.3125

203. The circuit shown in figure is made of a homogeneous wire of uniform cross-section. ABCD is
a square. Find the ratio of the amounts of heat liberated per unit time in wire A-B and C-D.
CURRENT ELECTRICITY 148

Ans. 11  6 2

204. A resistance coil of resistance r connected to an external battery, is placed inside an


adiabatic cylinder fitted with a frictionless piston of mass m and same area A. Initially
cylinder contains one mole of ideal gas He. A current I flows through the coil such that
temperature of gas varies as T  T0  at  bt 2 , keeping pressure constant with time t.
Atmosphere pressure above piston is P0 . Find
a) current I flowing through the coil as functions of time and
b) speed of piston as function of time.

1/2
 5R
Ans. a) I    2bt  a  b) V 
R
 2bt  a  where PA  P0 A  mg
 2r  PA

You might also like